Download as docx, pdf, or txt
Download as docx, pdf, or txt
You are on page 1of 89

CAMILO F. BORROMEO , petitioner, vs. ANTONIETTA O.

DESCALLAR,
respondent.

Facts: In 1984 in Cebu, Wilhelm Jambrich, an Austrian, met respondent, a separated


mother and waitress at that time. They fell in love and lived together. Later, they bought a
house and lots in Agro-Macro Subd, Mandaue City. In the Contracts to Sell, both of them
were referred to as the buyers of the Agro-Macro properties (1985 and 1986). A Deed of
Sale was executed in their favor (1987). However, the RD refused to register the Deed on
the ground that Jambrich was an alien and could not acquire alienable lands of the public
domain. Jambrich’s name was erased from the document, though his signature remained
on the left hand margin of page 1 beside Antonietta’s as buyer. The TCTs were issued in
Antonietta’s name alone.

They broke up in 1991.

In 1989, Jambrich became indebted to petitioner for P150,000.00. To pay for the debt, he
sold his rights and interests in the Agro-Macro properties to the petitioner thru a Deed of
Absolute Sale/Assignment for P250,000.00. However, in 1991, when petitioner sought to
register the Deed, he discovered that the titles to the three lots were transferred to
respondent, and that the property has already been mortgaged.

Petitioner filed a complaint against respondent for recovery of property before the RTC.
He claimed that the Deed of Absolute Sale over the properties do not reflect the true
agreement of the parties since respondent was not in fact the buyer, but Jambrich alone.
Respondent claimed that she solely and exclusively used her own personal funds for the
purchase, and that Jambrich, as an alien, was prohibited from acquiring such properties.

RTC: Ruled in favor of petitioner. It held that it is highly improbable that respondent
could acquire the properties while she was still working as a waitress earning P1,000/mo
salary as she could not even provide for the daily needs of her family. It also held that the
only probable reason why her name appeared in the contracts to sell was because as
observed by the Court, she is being a scheming and exploitative woman who took
advantage Jambrich’s goodness.

CA: Ruled in favor of respondent.

Issue: WON Jambrich, a foreigner, is the owner of said properties, making the transfer to
petitioner a valid transaction.

Ruling: Yes. Evidence clearly shows that it was he who had the financial capacity at
the time to acquire said properties, not respondent. Respondent likewise confirmed that
Jambrich was the owner of the properties but that his name was subsequently deleted
because of legal constraints, and that the money used for payment was in postdated
checks issued by Jambrich. Thus, Jambrich has all authority to transfer all his rights,
interests and participation over the subject properties to petitioner.Presumptions of co-
ownership and equal contribution do not apply. In such an adulterous relationship, no co-
ownership exists between the parties

The fact that the properties were registered in respondent’s name also does not
conclusively make her the owner. Registration is not a mode of acquiring ownership.
A certificate of title implies that the title is quiet, and that it is perfect, absolute and
indefeasible. However, there are exceptions to this rule, as when the transferee is not
a holder in good faith and did not acquire the subject properties for a valuable
consideration (as in this case).

The transfer of land to petitioner is valid, despite foreign citizenship of Jambrich. As held
in United Church Board for World Ministries, if land is invalidly transferred to an alien
who subsequently becomes a Filipino citizen or transfers it to a Filipino, the flaw in the
original transaction is considered cured and the title of the transferee is rendered valid.  

PETITION GRANTED.
ONG CHING PO, YU SIOK LIAN DAVID ONG and JIMMY ONG, petitioners, vs.
COURT OF APPEALS and SOLEDAD PARIAN, respondents.

Facts: Ong Joi Jong sold a parcel of land to Soledad Parian, wife of Ong Yee. Ong Yee
is the brother of petitioner Ong Ching Po. Both of them died subsequently. The sale
was evidenced in a notarized Deed of Sale which was registered in the RD in the
name of Parian, who said that she entrusted the administration of the lot and
building to petitioner Ong Ching Po. When her husband died, she demanded that the
lot be vacated because she was going to sell it. Petitioners refused to vacate the said
premises, claiming that their father, Ong Ching Po, bought the said land from Ong Joi
Jong and was conveyed to them.

Petitioners filed an action for reconveyance against respondent who, in turn, filed an
action for quieting of title against them.

RTC: In favor of respondent

CA: affirmed RTC’s decision

Petitioner Ong Ching Po claims that they have an agreement that the land be
registered in the name of respondent in order to avoid legal complications and to
facilitate registration and transfer and that the said title would be transferred by the
respondent to him or his successors-in-interest and that she would be holding the
title in trust for him.

Issue: WON an alien can acquire and own real property in the Philippines

Ruling: No. Ong Ching Po cannot claim that he merely used private respondent as a
dummy to have the title over the parcel of land registered in her name because
being an alien he was disqualified to own real property in the Philippines. The
capacity to acquire private land is made dependent upon the capacity to
acquire or hold lands of the public domain. Private land may be transferred or
conveyed only to individuals or entities "qualified to acquire lands of the
public domain." The 1935 Constitution reserved the right to participate in the
"disposition, exploitation, development and utilization" of all "lands of the public
domain and other natural resources of the Philippines" for Filipino citizens or
corporations at least 60% of the capital of which was owned by Filipinos. Aliens,
whether individuals or corporations, have been disqualified from acquiring public
lands; hence, they have also been disqualified from acquiring private lands. Ong
Ching Po was a Chinese citizen; therefore, he was disqualified from acquiring and
owning real property.

PETITION DISMISSED.
PHILIP MATTHEWS, petitioner, vs. BENJAMIN A. TAYLOR and JOSELYN C.
TAYLOR, respondents.

On 1988, respondent Benjamin, a British subject, married respondent Joselyn. On 1989,


Joselyn bought a lot in Boracay. The sale was allegedly financed by Benjamin. They used
the latter’s funds to construct improvements and converted the property to a resort.

Respondents had a falling out, and Joselyn ran away. On 1992, Joselyn executed an SPA
in favor of Benjamin, authorizing the latter to maintain, sell, lease, and sub-lease and
otherwise enter into contract with third parties with respect to their Boracay property.

Thereafter, Joselyn as lessor and petitioner Philip Matthews as lessee, entered into an
Agreement of Lease involving the Boracay property for a period of 25 years. Petitioner
took possession of the property and renamed the resort.

Claiming that the Agreement was null and void since as it was entered into by Joselyn
without Benjamin’s consent, Benjamin instituted an action for Declaration of Nullity of
Agreement of Lease with Damages against Joselyn and the petitioner. Benjamin claimed
that his funds were used in the acquisition and improvement of the Boracay property, and
coupled with the fact that he was Joselyn’s husband, any transaction involving said
property required his consent.

RTC: In favor of Benjamin. The RTC considered the Boracay property as their
community property. Thus, the consent of the spouses was necessary to validate any
contract involving the property.

CA: Affirmed RTC’s decision

Issue: WON the Lease Agreement of a parcel of land entered into by the Filipino wife
without the consent of her foreign husband is valid.

Ruling: Yes. Section 7, Article XII of the 1987 Constitution states that: Save in cases of
hereditary succession, no private lands shall be transferred or conveyed except to
individuals, corporations, or associations qualified to acquire or hold lands of the public
domain.

Benjamin has no right to nullify the Agreement of Lease between Joselyn and
petitioner. Benjamin, being an alien, is absolutely prohibited from acquiring private
and public lands in the Philippines. Considering that Joselyn appeared to be the
designated "vendee" in the Deed of Sale of said property, she acquired sole
ownership thereto. This is true even if Benjamin’s claim is sustained, that he provided
the funds for such acquisition. By entering into such contract knowing that it was illegal,
no implied trust was created in his favor; no reimbursement for his expenses can be
allowed; and no declaration can be made that the subject property was part of the
conjugal/community property of the spouses.
In any event, he had and has no capacity or personality to question the subsequent lease
of the Boracay property by his wife on the theory that in so doing, he was merely
exercising the prerogative of a husband in respect of conjugal property. To sustain such a
theory would countenance indirect controversion of the constitutional prohibition. If the
property were to be declared conjugal, this would accord the alien husband a substantial
interest and right over the land, as he would then have a decisive vote as to its transfer or
disposition. This is a right that the Constitution does not permit him to have.
CHARLES L. ONG, petitioner, vs. REPUBLIC OF THE PHILIPPINES, respondent.

Facts: Petitioner, in behalf of his brothers, filed for an application of Registration of


Title over Lot 15911. Petitioner alleged that he and his brothers are the co-owners
of the land, which they purchased from Sps. Bautista and Villamil and that their
predecessor-in-interest have been in open, continuous and peaceful possession of
the lot in the concept of owners for more than 30 years.

Only respondent, represented by the OSG, opposed the application for registration.
Respondent contends that neither applicants nor their predecessor-in-interest have
been in open, continuous, exclusive and notorious possession and occupation of the
lot since June 12, 1945 or earlier.

RTC: Favored Petitioner and declared the land in the name of the applicant.

CA: Reversed RTC’s ruling. petitioner failed to prove that he or his predecessors-in-
interest have been in adverse possession of the subject lot in the concept of owner
since June 12, 1945 or earlier as mandated by Section 14(1) of P.D. 1529. It noted
that the earliest tax declaration which petitioner presented is dated 1971.
Consequently, petitioner could not fairly claim possession of the land prior to 1971.
Neither was petitioner able to prove that he or his predecessors-ininterest actually
occupied the subject lot prior to the filing of the application.

Issue: WON possession alone is sufficient to acquire title to alienable lands of the
public domain.

Ruling: No. The law requires possession AND occupation. Pursuant to Section 14(1)
of P.D. 1529, applicants for registration of title must prove: (1) that the subject land
forms part of the disposable and alienable lands of the public domain, and (2) that
they have been in open, continuous, exclusive and notorious possession and
occupation of the same under a bona fide claim of ownership since June 12, 1945, or
earlier.

As held in Republic v. Alconaba: The law speaks of possession and occupation. Since
these words are separated by the conjunction and, the clear intention of the
law is not to make one synonymous with the other. Possession is broader than
occupation because it includes constructive possession. When, therefore, the
law adds the word occupation, it seeks to delimit all encompassing effect of
constructive possession. Taken together with the words open, continuous,
exclusive and notorious, the word occupation serves to highlight the fact that
for an applicant to qualify, his possession must not be a mere fiction. Actual
possession of a land consists in the manifestation of acts of dominion over it of such
a nature as a party would naturally exercise over his own property.

PETITION DENIED
REPUBLIC, petitioner, vs. CARLOS VEGA, MARCOS VEGA, ROGELIO VEGA, LUBIN
VEGA, HEIRS OF GLORIA VEGA, respondents,

ROMEA BUHAY-OCAMPO, FRANCISCO BUHAY, ARCELI BUHAY-RODRIGUEZ,


ORLANDO BUHAY, SOLEDAD BUHAY- VASQUEZ, LOIDA BUHAY-SENADOSA,
FLORENDO BUHAY, OSCAR BUHAY, ERLYN BUHAY-GINORGA, EVELYN BUHAY-
GRANETA, and EMILIE BUHAY-DALLAS, respondents-intervenors.

Facts: The respondents Vegas filed an application for registration of title covering a
parcel of land. They alleged that they inherited the subject land from their mother,
Maria, who in turn inherited it from her father, Lorenz.

The Republic filed an opposition to respondents Vegas’ application for registration


on the ground that the subject land or portions thereof were lands of the public
domain and, as such, not subject to private appropriation.

Respondents presented several exhibits in compliance with the jurisdictional


requirements, as well as witnesses to prove their ownership, occupation and
possession of the land subject of the registration. Significant was the testimony of
Mr. Gonzales, a Special Investigator of the CENRO of Los Bañ os, Laguna, under the
DENR. He attested to having conducted an inspection of the subject land and
identified the corresponding report which stated that the area subject of the
investigation was entirely within the alienable and disposable zone, and that there
was no public land application filed for the same land by the applicant or by any
other person.

Respondents intervenors Buhays entered their appearance and moved to intervene


in respondents Vegas’ application for registration. They claimed a portion of the
subject land consisting purportedly sold by respondents Vegas’ mother, Maria, to
the former’s predecessors in interest the sisters Gabriela and Isabel by virtue of a
Bilihan ng Isang Bahagi ng Lupang Katihan. They likewise formally offered in
evidence a Subdivision Plan, which indicated the portion of the subject land, which
they claimed was sold to their predecessors in interest.

RTC: granted respondents Vegas’ application and directed the LRA to issue the
corresponding decree of registration in the name of respondents Vegas and
respondents intervenors Buhays’ predecessors, in proportion to their claims over
the subject land.

The Republic appealed arguing that respondents Vegas failed to prove that the
subject land was alienable and disposable, since the testimony of Mr. Gonzales did
not contain the date when the land was declared as such.

CA: Affirmed RTC’s decision


Issue: WON respondents Vegas have sufficiently established that the subject land is
alienable and disposable

Ruling: The best proofs in registration proceedings that a land is alienable and
disposable are a certification from the CENRO or PENRO and a certified true copy of
the DENR’s original classification of the land. The Court, however, has nonetheless
recognized and affirmed applications for land registration on other substantial and
convincing evidence duly presented without any opposition from the LRA or the
DENR on the ground of substantial compliance. Applying these precedents, the
Court finds that despite the absence of a certification by the CENRO and a certified
true copy of the original classification by the DENR Secretary, there has been
substantial compliance with the requirement to show that the subject land is indeed
alienable and disposable based on the evidence on record.

Respondents Vegas were able to present Mr. Gonzales of the CENRO who testified .
Also, Subdivision Plan Csd formally offered as evidence by respondents intervenors
Buhays, indicates that the land is alienable and disposable. The LRA never raised the
issue that the land subject of registration was not alienable and disposable. In the
Supplementary Report submitted during the trial court proceedings, the LRA did not
interpose any objection to the application on the basis of the nature of the land.
HEIRS OF MARIO MALABANAN, petitioner, vs. REPUBLIC OF THE PHILIPPINES,
respondent.

Facts: On 1998, Mario Malabanan filed an application for land registration covering
a parcel of land. He claimed that he had purchased the property from Eduardo
Velazco, and that he and his predecessors-in-interest had been in open, notorious,
and continuous adverse and peaceful possession of the land for more than 30 years.
Velazco testified that the property was originally belonged to a twenty-two hectare
property owned by his great-grandfather, Lino Velazco. Lino had four sons–
Benedicto, Gregorio, Eduardo and Esteban. Upon Lino’s death, his four sons
inherited the property and divided it among themselves. But by 1966, Esteban’s
wife, Magdalena, had become the administrator of all the properties inherited by the
Velazco sons. After the death of Esteban and Magdalena, their son Virgilio succeeded
them in administering the properties, including Lot 9864-A, which originally
belonged to his uncle Eduardo. It was this property that was sold by Eduardo to
Malabanan.

Among the evidence presented by Malabanan during trial was a Certification dated
in 2001, issued by the CENRO-DENR, which stated that the subject property was
verified to be within the Alienable or Disposable land

RTC: Approved the application for registration.

CA: Reversed the RTC ruling and dismissed the application of Malabanan.

ISSUES: WON petitioners can register through Section 14(2) of the Property
Registration Decree and therefore susceptible to acquisition by prescription in
accordance with the Civil Code

Ruling: No. In complying with Section 14(2) of the Property Registration Decree,
consider that under the Civil Code, prescription is recognized as a mode of acquiring
ownership of patrimonial property. However, public domain lands become only
patrimonial property not only with a declaration that these are alienable or
disposable. There must also be an express government manifestation that the
property is already patrimonial or no longer retained for public service or the
development of national wealth, under Article 422 of the Civil Code. And only when
the property has become patrimonial can the prescriptive period for the acquisition
of property of the public dominion begin to run.

Patrimonial property is private property of the government. The person acquires


ownership of patrimonial property by prescription under the Civil Code is entitled
to secure registration thereof under Section 14(2) of the Property Registration
Decree.
There are two kinds of prescription by which patrimonial property may be acquired,
one ordinary and other extraordinary. Under ordinary acquisitive prescription, a
person acquires ownership of a patrimonial property through possession for at least
10 years, in good faith and with just title. Under extraordinary acquisitive
prescription, a person’s uninterrupted adverse possession of patrimonial property
for at least 30 years, regardless of good faith or just title, ripens into ownership.

It is clear that the evidence of petitioners is insufficient to establish that Malabanan


has acquired ownership over the subject property under Section 48(b) of the Public
Land Act. Also, there is no substantive evidence to establish that Malabanan or
petitioners as his predecessors-in-interest have been in possession of the property
since 12 June 1945 or earlier. The earliest that petitioners can date back their
possession, according to their own evidence—the Tax Declarations they presented
in particular—is to the year 1948. Thus, they cannot avail themselves of registration
under Section 14(1) of the Property Registration Decree.

Neither can petitioners properly invoke Section 14(2) as basis for registration.
While the subject property was declared as alienable or disposable in 1982, there is
no competent evidence that is no longer intended for public use service or for the
development of the national evidence, conformably with Article 422 of the Civil
Code. The classification of the subject property as alienable and disposable land of
the public domain does not change its status as property of the public dominion
under Article 420(2) of the Civil Code. Thus, it is insusceptible to acquisition by
prescription.

PETITION DENIED.
HEIRS OF MALABANAN V. REPUBLIC
GR NO. 179987. SEPTEMBER 3, 2013

On 20 February 1998, Mario Malabanan filed an application for land registration before the
RTC of Cavite-Tagaytay, covering a parcel of land situated in Silang Cavite. Malabanan
claimed that he had purchased the property from Eduardo Velazco, and that he and his
predecessors-in-interest had been in open, notorious, and continuous adverse and peaceful
possession of the land for more than thirty (30) years. Velazco testified that the property
was originally belonged to a twenty-two hectare property owned by his great-grandfather,
Lino Velazco. Upon Lino’s death, his four sons inherited the property and divided it among
themselves. But by 1966, Esteban’s wife, Magdalena, had become the administrator of all
the properties inherited by the Velazco sons from their father, Lino. After the death of
Esteban and Magdalena, their son Virgilio succeeded them in administering the properties,
including Lot 9864-A, which originally belonged to his uncle, Eduardo Velazco. It was this
property that was sold by Eduardo Velazco to Malabanan.

Among the evidence presented by Malabanan during trial was a Certification Issued by the
Community Environment & Natural Resources Office, Department of Environment and
Natural Resources (CENRO-DENR), which stated that the subject property was “verified to
be within the Alienable or Disposable land per Land Classification Map No. 3013 established
under Project No. 20-A and approved as such under FAO 4-1656 on March 15, 1982.”

RTC: Approved the application for registration.

The Republic interposed an appeal to the Court of Appeals, arguing that Malabanan had
failed to prove that the property belonged to the alienable and disposable land of the public
domain, and that the RTC had erred in finding that he had been in possession of the
property in the manner and for the length of time required by law for confirmation of
imperfect title.

CA: Reversed the RTC ruling and dismissed the application of Malabanan.

Issues: W/N the character of petitioner’s possession and occupation of the subject property
entitles them to own the land.

SC: No, since it did not meet the requirements of the law. Petition is denied.

There are two laws that should be discussed here. Sections 14(1) of PD 1529 in connection
with 48(b) of the Public Land Act and Section (2) of PD 1529 in connection with Art 422 of
the Civil Code.

1. In connection with Section 14(1) of the Property Registration Decree, Section 48(b)
of the Public Land Act recognizes and confirms that “those who by themselves or through
their predecessors in interest have been in open, continuous, exclusive, and notorious
possession and occupation of alienable and disposable lands of the public domain, under a
bona fide claim of acquisition of ownership, since June 12, 1945” have acquired ownership
of, and registrable title to, such lands based on the length and quality of their possession.
(a) Since Section 48(b) merely requires possession since 12 June 1945 and does not
require that the lands should have been alienable and disposable during the
entire period of possession, the possessor is entitled to secure judicial
confirmation of his title thereto as soon as it is declared alienable and disposable,
subject to the timeframe imposed by Section 47 of the Public Land Act; and
(b) The right to register granted under Section 48(b) of the Public Land Act is further
confirmed by Section 14(1) of the Property Registration Decree.

2. In complying with Section 14(2) of the Property Registration Decree, consider that
under the Civil Code, prescription is recognized as a mode of acquiring ownership of
patrimonial property. However, public domain lands become only patrimonial property
not only with a declaration that these are alienable or disposable. There must also be an
express government manifestation that the property is already patrimonial or no
longer retained for public service or the development of national wealth, under Article
422 of the Civil Code. And only when the property has become patrimonial can the
prescriptive period for the acquisition of property of the public dominion begin to run.

A. Patrimonial property is private property of the government. The person acquires


ownership of patrimonial property by prescription under the Civil Code is entitled to
secure registration thereof under Section 14(2) of the Property Registration Decree.
B. There are two kinds of prescription by which patrimonial property may be
acquired, one ordinary and other extraordinary. Under ordinary acquisitive
prescription, a person acquires ownership of a patrimonial property through possession
for at least ten (10) years, in good faith and with just title. Under extraordinary
acquisitive prescription, a person’s uninterrupted adverse possession of
patrimonial property for at least thirty (30) years, regardless of good faith or just
title, ripens into ownership.

It is clear that the evidence of petitioners is insufficient to establish that Malabanan has
acquired ownership over the subject property under Section 48(b) of the Public Land Act.
There is no substantive evidence to establish that Malabanan or petitioners as his
predecessors-in-interest have been in possession of the property since 12 June 1945 or
earlier. The earliest that petitioners can date back their possession, according to their own
evidence—the Tax Declarations they presented in particular—is to the year 1948. Thus,
they cannot avail themselves of registration under Section 14(1) of the Property
Registration Decree.

Neither can petitioners properly invoke Section 14(2) as basis for registration. While the
subject property was declared as alienable or disposable in 1982, there is no competent
evidence that is no longer intended for public use service or for the development of the
national evidence, conformably with Article 422 of the Civil Code. The classification of the
subject property as alienable and disposable land of the public domain does not change its
status as property of the public dominion under Article 420(2) of the Civil Code. Thus, it is
insusceptible to acquisition by prescription.

REPUBLIC OF THE PHILIPPINES V. METRO INDEX REALTY AND DEVELOPMENT


CORPORATION
GR NO 198585. JULY 2, 2012

Respondent Metro Index Realty and Development Corporation filed with the Regional Trial
Court (RTC), an application for judicial confirmation of title over three (3) parcels of land
located at Barangay Alulod/Mataas na Lupa, Indang, Cavite.

At the trial, respondent presented 2 witnesses, which testified the following: Enrico, who
was the respondent's Project Documentation Officer, 􏰃said that: (a) the respondent bought
the subject properties from Sicap family; (b) the subject properties had been declared for
tax purposes in the respondent's name since 2006; (c) the subject properties are alienable
and disposable as evidenced by the certification issued by the Department of Environment
and Natural Resources (DENR); (d) as shown by their respective affidavits, the adjoining lot
owners had no adverse claim and objections to the respondent's application; and (e) the
respondent and its predecessors-in-interest had been in possession of the subject
properties for more than fifty (50) years. Herminia, on the other hand, testified that: (a) she
and her siblings, Melinda and Hernando, inherited the subject properties from their parents,
Brigido Sicap and Juana Espineli; (b) their parents had been in possession of the subject
properties since 1956 as shown by the tax declarations in their name; (c) from the time they
inherited the subject properties, they had actively cultivated them and religiously paid the
taxes due; 2 and (d) the subject properties are planted with coconut, banana, santol, palay
and corn.

Issue: W/N respondents can own the property through prescription.

RTC: Granted respondent’s petition. Land is alienable and disposable, and that the
possession of herein applicant as well as that of its predecessor(s)-in-interest has (sic) been
open, public[,] continuous, notorious and adverse to the whole world

Petitioner interposed: Respondent failed to prove compliance with the requirements of


Sec 14(1) or (2) of PF 1529 that their possession and occupation commenced on June 12,
1945 or earlier. Their earliest tax declaration was only from 1956. Respondent also failed to
show proof of an official declaration that the subject properties are no longer intended for
public service or for the development of national wealth; hence, the subject properties
cannot be acquired by prescription.

CA: Denied the petition. Affirmed RTC’s decision. While only a few trees are found on the
subject properties, this fact coupled with the diligent payment of taxes since 1956 sufficed
to substantiate the claim that the respondent and its predecessors-in-interest had been in
possession in the manner and for the length of time required by law.

SC: The Court grants the petition to deny respondent’s application for original certification
of the land in question. That properties of the public dominion are not susceptible to
prescription and that only properties of the State that are no longer earmarked for public
use, otherwise known as patrimonial, may be acquired by prescription are fundamental,
even elementary, principles in this jurisdiction.

According to Sec 14(2) of PD 1529, the following persons may 􏰃file in the proper Court of
First Instance an application for registration of title to land, whether personally or through
their duly authorized representatives:
Xxxx xxx xxx
(2) Those who have acquired ownership of private lands by prescription under the
provisions of existing laws.
Following said law, we have declared that:
(A) the prescriptive period for purposes of acquiring an imperfect title over a property of
the State shall commence to run from the date an official declaration is issued that
such property is no longer intended for public service or the development of
national wealth; and
(B) Prescription will not run as against the State even if the property has been
previously classified as alienable and disposable as it is that official declaration that
converts the property to patrimonial. Particularly:

In complying with Section 14(2) of the Property Registration Decree, consider that
under the Civil Code, prescription is recognized as a mode of acquiring ownership of
patrimonial property. However, public domain lands become only patrimonial property not
only with a declaration that these are alienable and disposable. There must also be an
express government manifestation that the property is already patrimonial or no
longer retained for public service or the development of national wealth, under
Article 422 of the Civil Code. And only when the property has become patrimonial can the
prescriptive period for the acquisition of property of the public dominion begin to run.

It is not the notorious, exclusive and uninterrupted possession and occupation of an


alienable and disposable public land for the mandated periods that converts it to
patrimonial, but the declaration that the property is already patrimonial or is no longer
retained for public service. This Court 􏰃finds no evidence of such official declaration and for
this reason alone, the respondent's application should have been dismissed outright.

Also, rather than proof of constructive possession, the presence of a meager number of
plantings on the subject properties shows that the respondent and its predecessors- in-
interest engaged in mere casual cultivation, which does not constitute possession under
claim of ownership. The evidence on the planted fruits and crops merely showed casual or
occasional cultivation of portions of the land in question. In short, possession is not
exclusive nor notorious, much less continuous, so as to give rise to a presumptive grant
from the government.
REPUBLIC VS RIZALRO, JR
GR NO. 1720111. MARCH 7, 2011

Respondent filed before the MTC of Bauag, La Union an application for the registration of a
parcel of land located on Bauag, La Union. He alleged that he is the owner in fee simple of
the subject parcel of land, that he obtained title over the land by virtue of a Deed of Transfer
which he purchased from his mother, and that he was in adverse, open, exclusive and
notorious possession of the land. He presented a Tax Declaration for 1994 and Proof of
Payment of real property taxes beginning 1952 up to the time of filing of the application.
Respondent’s mother, Bibiana, also presented during trial the Absolute Deed of Sale date
July 8, 1952 which evidenced her purchase of lot from Navarro. Respondent Rizalro Jr. also
stated that he was not aware of any person or entity which questioned his mother's
ownership and possession of the subject property.

The OSG filed an Opposition alleging that:


1. Neither respondent nor his predecessors-in-interest had been in open, continuous,
exclusive and notorious possession and occupation of the subject property since June
12, 1945 or earlier;
2. There was no clear acts off possession and occupation such as fencing, putting a wall,
cultivating or otherwise on the said land;
3. That the tax declarations and tax payment receipts did not constitute competent and
sufficient evidence of ownership; and
4. That the subject property was a portion of public domain belonging to the Republic of
the Philippines and hence not subject to private acquisition.

Community Environment and Natural Resources Office (CENRO) conducted an investigation


and verification of the records of the subject land and certified that indeed the subject
parcel of land was within the alienable and disposable zone and that the applicant was
indeed in actual occupation and possession of the land.

MTC: Approved respondent’s application.

CA: Petitioner appealed but CA found no merit and promulgated the assailed decision,
affirming the trial court’s decision.

Issue: W/N respondent’s application to own the land should be granted.

SC: We answer in the negative. Respondent’s application for registration is denied.

Under Section 14 (1) of PD 1529, applicants for registration of title must sufficiently
establish:
1. First, that the subject land forms part of the disposable and alienable lands of the public
domain;
5. Second, that the applicant and his predecessors-in-interest have been in open,
continuous, exclusive and notorious possession and occupation of the same; and
6. Third, that it is under a bona fide claim of ownership since June 12, 1945, or earlier.

The first requirement was satisfied in this case. The certification and report of the CENRO of
San Fernando City, La Union, states that the entire land area in question is within the
alienable and disposable zone. This certification enjoys the presumption of regularity and is
sufficient proof to show the classification of the land described herein.

Respondent has likewise met the second requirement as to ownership and possession. The
MTC and the CA both agreed that respondent has presented sufficient testimonial and
documentary evidence to show that he and his predecessors-in-interest were in open,
continuous, exclusive and notorious possession and occupation of the land in question.

However, the third requirement, that respondent and his predecessors-in-interest be in


open, continuous, exclusive and notorious possession and occupation of the subject
property since June 12, 1945 or earlier, has not been satis􏰆ed. Respondent only managed
to present oral and documentary evidence of his and his mother's ownership and
possession of the land since 1952, and although he presented a tax declaration in the name
of the former owner, Navarro, the same was only on 1948. Thus, the same fails to comply
with the requirements of the law.

It is jurisprudentially clear that the thirty (30)-year period of prescription for purposes of
acquiring ownership and registration of public land under Section 14 (2) of P.D. No. 1529
only begins from the moment the State expressly declares that the public dominion
property is no longer intended for public service or the development of the national
wealth or that the property has been converted into patrimonial. In the case at bar,
respondent merely presented a certification and report from the DENR-CENRO dated July
17, 2001 certifying that the land in question entirely falls within the alienable and
disposable zone since January 21, 1987; that it has not been earmarked for public use; and
that it does not encroach any area devoted to general public use. Unfortunately, such
certification and report is not enough in order to commence the thirty (30)-year
prescriptive period under Section 14 (2), and even assuming it is enough, mere 13 years
after is far too short of the required 30 years under existing laws.
REPUBLIC V. ABRILLE
GR NO L-39248. MAY 7, 1976

Luisa Abrille owns a lot under certificate of title T-1439 which was later subdivided into T-
18886 and T-18887. The lot T- 18886 was old to Gaudencio Consunji, while T-18887 was to
be distributed amongst her heirs. However, prior to registration of T- 18887, the lot
increased in size because the river dried up. But despite the increase of the land area, the
Registry of Deeds still issued a new certificate of title for the subdivision.

Hence, Director of Lands and the Republic filed for the annulment of such certificates as it
was not in accordance with law since the enlarged portion of the land in question was
formerly part of the Davao River which dried up by reason of the change of course of the
river; hence the land belongs to the public domain.

Issue: W/N a TCT can be annulled considering the fact that the lot increased due to the
drying up of the river?

RTC: Decided In favor of DIRECTOR OF LANDS and cancelled the new certificates of the
titled land containing the increased area and ordered the RD to issue new ones in lieu
thereof After the increased portion shall have been segregated therefrom in accordance
with law.

CA: Certified the case to the SC since it involved purely a question of law.

SC: We affirm the decisions of the lower court. In order to bring this increase in area, which
the parties admitted to have been a former river bed of the Davao River, under the
operation and coverage of the Land Registration Law, Act 496, proceedings in
registrations of land title should have been filed Instead of an ordinary approval of
subdivision plan.

For an applicant to have his imperfect or incomplete title or claim to a land to be originally
registered under Act 496, the following requisites should all be satisfied:
1. Survey of land by the Bureau of Lands or a duly licensed private surveyor;
7. Filing of application for registration by the applicant;
8. Setting of the date for the initial hearing of the application by the Court;
9. Transmittal of the application and the date of initial hearing together with all the
documents or other evidences attached thereto by the Clerk of Court to the Land
Registration Commission;
10. Publication of a notice of the filing of the application and date and place of the hearing in
the Official Gazette;
11. Service of notice upon contiguous owners, occupants and those known to have interests
in the property by the sheriff;
12. Filing of answer to the application by any person whether named in the notice or not;
13. Hearing of the case by the Court;
14. Promulgation of judgment by the Court;
15. Issuance of the decree by the Court declaring the decision final and instructing the Land
Registration Commission to issue a decree of confirmation and registration;
16. Entry of the decree of registration in the Land Registration Commission;
17. Sending of copy of the decree of registration to the corresponding Register of Deeds,
and
18. Transcription of the decree of registration in the registration book and the issuance of
the owner's duplicate original certificate of title to the applicant by the Register of
Deeds, upon payment of the prescribed fees.

In the instant case, part of the tracts of land, particularly the area of 82,127 square meters,
has not yet been brought under the operation of the Torrens System. Worse still, the
approval of Subdivision Plans was without notice to all parties in interest, more particularly
the Director of Lands.

Hence, with the foregoing requisites not having been complied with, the lower court
committed no error in its appealed decision. Decision is affirmed in toto.
OFFICE OF THE CITY MAYOR OF PARANAQUE V. EBIO
GR NO 178411. JUNE 23, 2010

Respondents claim that they own the parcel of land locate din Brgy. Vitalez, Paranaque City
which is covered by a Tax declaration in the name of Mario Ebio. Respondents claim that
said land was an accretion of Cut-cut creek. Respondents assert that the original occupant
and possessor of the said parcel of land was their great grandfather, Jose Vitalez. Sometime
in 1930, Jose gave the land to his son, Pedro Vitalez. From then on, Pedro continuously and
exclusively occupied and possessed the said lot. In 1966, after executing an affidavit
declaring possession and occupancy, Pedro was able to obtain a tax declaration over the
said property in his name. Since then, respondents have been religiously paying real
property taxes for the said property. In 1961, Mario established their home on the said lot
which is secured by a building permit. Pedro then executed a notarized Transfer of Rights
ceding his ownership to Mario, which subsequently, made the tax declaration be cancelled
and a new one be issued in the name of Mario.

Petitioner on 1999 planned to construct an access road along the cut-out creek that will
traverse the lot occupied by the respondents. When the city government advised all the
affected residents to vacate the said area, respondents immediately registered their
opposition thereto. As a result, the road project was temporarily suspended. On March 28,
2005, City Administrator Noli Aldip sent a letter to the respondents ordering them to vacate
the area within the next thirty (30) days, or be physically evicted from the said property.

Threatened of being evicted, respondents went to the RTC of Parañ aque City on April 21,
2005 and applied for a writ of preliminary injunction against petitioners.

RTC: The RTC issued an order denying the petition for lack of merit. The trial court
reasoned that respondents were not able to prove successfully that they have an
established right to the property since they have not instituted an action for confirmation of
title and their application for sales patent has not yet been granted.

CA: Court of Appeals reversed and set aside RTC’s decision and issued its Decision in favor
of the respondents. The evidentiary records of the instant case, shows that RL 8 containing
an area of 291 square meters is owned by Guaranteed Homes, Inc. covered by TCT No. S-
62176. The same RL 8 appears to have been donated by the Guaranteed Homes to the City
Government of Parañ aque. However, It is clear that since 1930, Appellants together with
their predecessor-in-interest, PEDRO VITALEZ, have been in exclusive possession of the
subject property and starting 1964 had introduced improvements thereon as evidenced by
their construction permits. Thus, even by extraordinary acquisitive prescription, Appellants
have acquired ownership of the property in question since 1930 even if the adjoining RL 8
was subsequently registered in the name of Guaranteed Homes. Further, it was only in
1978 that Guaranteed Homes was able to have RL 8 registered in its name, which is
almost 􏰈fifty years from the time PEDRO VITALEZ occupied the adjoining accreted
property in 1930.

Issue: W/N respondent’s possession and occupation of the subject property entitles them
to avail of the relief of prohibitory injunction.
SC: We find that the character of possession and ownership by the respondents over the
contested land entitles them to the avails of the action. Thus, petition is denied, CA’s
decision is affirmed.

It is an uncontested fact that the subject land was formed from the alluvial deposits that
have gradually settled along the banks of Cut-cut creek. This being the case, the law that
governs ownership over the accreted portion is Article 84 of the Spanish Law of Waters of
1866, which remains in effect, in relation to Article 457 of the Civil Code. It is explicit from
the foregoing provisions that alluvial deposits along the banks of a creek do not form part of
the public domain as the alluvial property automatically belongs to the owner of the estate
to which it may have been added. Hence, while it is true that a creek is a property of public
dominion, the land which is formed by the gradual and imperceptible accumulation of
sediments along its banks does not form part of the public domain by clear provision of law.

Undoubtedly, respondents are deemed to have acquired ownership over the subject
property through prescription. Respondents can assert such right despite the fact that they
have yet to register their title over the said lot. It must be remembered that the purpose of
land registration is not the acquisition of lands, but only the registration of title which the
applicant already possessed over the land.
REPUBLIC V. SANTOS III
GR NO 160453. November 12, 2012

Respondent Ivan Arcadio applied for the registration of the subject land in the RTC.
Respondents allege that they have been in possession of the land in question for more that
10 years and that the property has been formed through accretion and had been in their
joint open, notorious, public, continuous and adverse possession for more than 30 years.

City of Paranaque opposed the application for land registration, stating that:
1. It needed the property for its flood control program;
19. That the property was within the legal easement of 20 meters from the river bank; and
20. That the property was an orchard that had dried up and had not resulted from
accretion.

RTC: Granted the application of the land registration.

CA: Affirmed the decision of the RTC

Issue: W/N respondent could claim the property by virtue of acquisitive prescription
pursuant to Sec 14 (1) of PD 1529

SC: No, they could not. This Court reverses and sets aside CA’s decision. We declare the
subject land as exclusively belonging to the State for being part of the dried-up bed of the
Paranaque river.

Evidence revealed that the property was the dried-up river bed of the Parañ aque River and
not an accretion of land, even both the RTC and the CA to themselves hold that said land
was "the land which was previously part of the Parañ aque River . . . (and) became an
orchard after it dried up." Respondents did not establish at all that the increment of land
had formed from the gradual and imperceptible deposit of soil by the effects of the current.
Also, it seems to be highly improbable that the large volume of soil that ultimately
comprised the dry land with an area of 1,045 square meters had been deposited in a
gradual and imperceptible manner by the current of the river in the span of about 20 to 30
years.

Thus, the RTC and the CA grossly erred in treating the dried-up river bed as an accretion
that became respondents' property pursuant to Article 457 of the Civil Code. The State
exclusively owned the said lot and may not be divested of its right of ownership.

Also, under Section 14 (1), then, applicants for confirmation of imperfect title must prove
the following, namely:
1. That the land forms part of the disposable and alienable agricultural lands of the public
domain; and
21. That they have been in open, continuous, exclusive, and notorious possession and
occupation of the land under a bona fide claim of ownership either since time
immemorial or since June 12, 1945

The first requirement was not met. To prove that the land subject of an application for
registration is alienable, an applicant must conclusively establish the existence of a positive
act of the Government. In this case, respondent did not submit proof that the and was
already declared as alienable and disposable by the Government. Absent such declaration,
the land still belonged to the State.

The second requirement was not also met. The admission of respondents themselves that
they declared the property for taxation purposes only in 1997 and paid realty taxes only
from 1999 28 signified that their alleged possession would at most be for only nine years as
of the filing of their application for land registration on March 7, 1997.
Heirs of Narvasa Sr. vs. Imbornal, G.R. No. 182908, Aug. 6, 2014

During her lifetime, Basilia owned a parcel of land situated at Sabangan, Barangay
Nibaliw West, San Fabian, Pangasinan (Sabangan property), which she conveyed to her
three (3) daughters Balbina, Alejandra, and Catalina (Imbornal sisters) sometime in 1920
Meanwhile, Catalina’s husband, Ciriaco Abrio (Ciriaco), applied for and was granted a
homestead patent over a riparian land (Motherland) adjacent to the Cayanga River in San
Fabian, Pangasinan. Ciriaco and his heirs had since occupied the northern portion of the
Motherland while respondents occupied the southern portion.

The First Accretion adjoined the southern portion of the Motherland. OCT therefor was
issued in the name of respondent Victoriano, married to Esperanza Narvarte, covering the
First Accretion. Decades later, the Second Accretion abutted the First Accretion on its
southern portion. OCT therefor was issued in the names of all the respondents covering
the Second Accretion.

Claiming rights over the entire Motherland, Francisco, et al., as the children of Alejandra
and Balbina filed an Amended Complaint for reconveyance, partition, and/or damages
against respondents.They anchored their claim on the allegation that Ciriaco, with the
help of his wife Catalina, urged Balbina and Alejandra to sell the Sabangan property, and
that Ciriaco used the proceeds therefrom to fund his then-pending homestead patent
application over the Motherland. In return, Ciriaco agreed that once his homestead patent
is approved, he will be deemed to be holding the Motherland — which now included both
accretions — in trust for the Imbornal sisters.

Francisco et al. argues that: through deceit, fraud, falsehood, and misrepresentation,
respondent Victoriano, with respect to the First Accretion, and the respondents
collectively, with regard to the Second Accretion, had illegally registered the said
accretions in their names, notwithstanding the fact that they were not the riparian owners
(as they did not own the Motherland to which the accretions merely formed adjacent to).

Thus, bewailing that respondents have refused them their rights not only with respect to
the Motherland, but also to the subsequent accretions, Francisco, et al. prayed for the
reconveyance ofsaid properties, or, in the alternative, the payment of their value.

RTC ruled in favor of Petitioner. With respect to the accretions that formed adjacent to
the Motherland, the RTC ruled that the owner of the Motherland is likewise the owner of
the said accretions. Considering that the Imbornal sisters have become proportionate
owners of the Motherland by virtue of the implied trust created between them and
Ciriaco, they (Imbornal sisters) and their heirs are also entitled to the ownership of said
accretions despite the fact that respondents were able to register them in their names.

CA reversed and ruled in favor of the respondents. CA ruled that respondents — i.e.,
respondent Victoriano with respect to the First Accretion, and all the respondents with
respect to the Second Accretion — need not be the owners of the Motherland in order to
acquire them by acquisitive prescription. Considering that accretions are not
automatically registered in the name of the riparian owner and are, therefore, subject to
acquisitive prescription by third persons, any occupant may apply for their registration.

(ISSUE: who has better right over the first accretion and second accretion)

SC ruled that the Imbornals have the better right. Since Francisco et al. failed to prove
their ownership rights over the Motherland, their cause of action with respect to the First
Accretion and, necessarily, the Second Accretion, must likewise fail.
Susi vs. Razon, G.R. No. 24066, December 9, 1925

Petitioner has been in open, continuous, exclusive and notorious possession and
occupation of a land since 1899. He was in peaceful and uninterrupted possession and
occupation of the land until Razon, in 1913, sued to recover the land from Suzi. CFI ruled
in favor of Susi. Razon, desperate, applied for the purchase of the land from the Director
of Lands, which the latter approved. Razon, now with a certificate of title, sued for
forcible entry against Susi.

Doctrine: Open, continuous, exclusive and notorious possession of alienable public land
for the period prescribed by law creates the legal fiction whereby the land, upon
completion of the requisite period, ipso jure and without the need of judicial or other
sanction, ceases to be public land and becomes private land. When Razon applied for the
purchase of the land, Susi had already been in possession thereof personally for more
than 30 years. Given that, Susi had already acquired, by operation of law, not only a right
to a grant, but a grant of the Government, for it is not necessary that certificate of title
should be issued in order that said grant may be sanctioned by the courts, an application
therefor being sufficient

Republic vs. Noval, G.R. No. 170316, Sept. 18, 2017

On September 8, 1999, the applicants sought the registration of their titles over the
subdivided portions of a land in Barangay Casili, Consolacion, Cebu. They alleged to
have acquired their respective portions of this land by "purchase, coupled with
continuous, public, notorious, exclusive and peaceful possession in the concept of an
owner for more than 30 years including the possession of their predecessors-in-interest."
They also alleged that they were in actual possession of their respective portions of the
property. While the Department of Environment and Natural Resources did not issue a
certification, it did approve their survey plan when the property was partitioned.

The Republic through the Office of the Solicitor General filed its Opposition on the
ground that the applicants failed to prove open, continuous, exclusive, and notorious
possession of the property since June 12, 1945. It also argued that the property sought
to be registered was part of the public domain. The tax declarations and tax payment
receipts attached to the application were not competent to show bona fide acquisition or
open and continuous possession of the land. There was no Department ofEnvironment
and Natural Resources report submitted to show when the properties were declared
alienable and disposable, for the purpose of computing the 30-year period of possession
required by law. Respondents' and their predecessor-in-interest's possession as mere
casual cultivation.

The applicants’ predecessor-in-interest was Cecilia Quindao (Cecilia), who was already
73 years old when she testified that her grandmother had possessed the said land. This
was inherited by her father who also cultivated the land and planted crops, the fruits of
which they had enjoyed. When her father died, she inherited the land and sold it to Joel
Noval (Joel) and Elizabeth Messerli (Messerli).
The MTC granted their application for registration of title. It declared the applicants to be
the absolute owners and possessors of their respective lots, having established
conclusively that they are the exclusive owners and peaceful possessors of the properties.

The Republic appealed the Decision of the trial court, arguing that: the applicants failed
to show open, continuous, exclusive, and notorious possession of alienable and
disposable lands for 30 years; it reiterated that tax declarations may not be used as bases
for the grant of the application; it added that there was no Department of Environment
and Natural Resources report submitted to show when the properties were declared
alienable and disposable, for the purpose of computing the 30-year period of possession
required by law.CA nevertheless affirmed MTC’s decision. MR was likewise denied.
Hence, this petition.
ISSUE: WON whether or not the Court of Appeals erred in affirming the trial court
decision.

SC: Petition denied. Under the Public Land Act, public lands may be disposed of through
confirmation of imperfect or incomplete titles. Confirmation of title may be done
judicially or through the issuance of a free patent. The process for judicial confirmation
of title is outlined in Section 48 of the Public Land Act, as amended by Presidential
Decree No. 1073

When a person applies for judicial confirmation of title, he or she already holds an
incomplete or imperfect title over the property being applied for, after having been in
open, continuous, exclusive, and notorious possession and occupation from June 12, 1945
or earlier. The date "June 12, 1945" is the reckoning date of the applicant's possession
and occupation, and not the reckoning date of when the property was classified as
alienable and disposable.

Thus, a property applied for judicial confirmation of title may be classified as alienable
and disposable at any time. For the purposes of judicial confirmation of title, only
possession and occupation must be reckoned from June 12, 1945.
Applicants for judicial confirmation of title must still comply with the requisites stated in
Section 48 (b) of the Public Land Act and Section 14 (1) of the Property Registration
Decree:
1. The applicant, by himself or through his predecessor-in-interest, has been in
possession and occupation of the property subject of the application;
2. The possession and occupation must be open, continuous, exclusive, and
notorious;
3. The possession and occupation must be under a bona fide claim of acquisition of
ownership;
4. The possession and occupation must have taken place since June 12, 1945, or
earlier; and
5. The property subject of the application must be an agricultural land of the public
domain.
These have been complied by the applicants.
Republic vs. IAC and ACME Plywood and Veneer Co., G.R. No. 73002, Dec. 29,
1986

There was an order for registration of five parcels of land acquired by ACME Plywood
from Mariano and Acer Infiel, who are members of the Dumagat tribe. The Director of
Lands filed an appeal for certiorari because the prevailing law then was the prohibition
against private corporations from holding lands of public domain except in lease not
exceeding 1,000 hectares.

Doctrine: Where at the time the corporation acquired the land, its predecessors-in-interest
had been in possession and occupation thereof in the manner and for the period
prescribed by law as to entitle him to registration in his name, then the proscription
against corporations acquiring alienable lands of the public domain except through lease
does not apply for the land was no longer public land but private property.

Alienable public land held by a possessor, personally or through his predecessors in


interest, openly, continuously and exclusively for the prescribed statutory period (30
years) is converted to private property by mere lapse or completion of said period.
Following that rule, the subject land was already private property at the time it was
acquired from the Infiels. Acme thereby acquired a registrable title, there being at the
time no prohibition against said corporation’s holding or owning private land. (Comment:
RA 1942 was the law existing at this time
Republic vs. Rovancy Realty and Development Corp. G.R. No. 190817, Jan. 10, 2018

RRDC filed before the RTC an Amended Application for Registration covering a parcel
of land identified as (subject land) situated in Barangay Balulang, Cagayan de Oro City.

RRDC alleged, among others, that it is a domestic corporation duly organized and
existing under and by virtue of the laws of the Republic of the Philippines; that it is the
absolute owner in fee simple of the subject land having acquired the same from its
previous owner, P.N. Roa Enterprises, Inc., by virtue of a notarized deed of absolute sale.

An opposition to the application was filed by the Heirs of Paulino Avanceña. They
alleged, that the subject land was already claimed and owned by the late Atty. Paulino
Avanceña (Paulino), their father and predecessor-in-interest, as early as 1926; that
Paulino had been in open, continuous, notorious, adverse, and exclusive possession and
occupation of the subject land; that Paulino registered the subject land for taxation
purposes and has paid the taxes due thereon in 1948; that their parents, Paulino and
Rizalina Neri (Rizalina) merely allowed and tolerated Pedro N. Roa's (Pedro) possession
of the subject land after the latter approached them and requested that he be allowed to
use the subject land for his businesses; that Pedro is one of RRDC's predecessors-in-
interest; that sometime in 1994, Rizalina demanded the return of the subject land from the
heirs of Pedro, but to no avail; that in 1996, Rizalina died leaving the private oppositors
as the rightful heirs of the subject land; that their parents never sold the subject land to
Pedro nor to RRDC, and as such, no right or title over the subject land was passed on to
RRDC. Thus, they prayed that RRDC's application be dismissed, and that their
opposition be treated as their own application for registration.

Republic of the Philippines (Republic), through (OSG), filed its opposition to the
application on the following grounds: that neither RRDC nor its predecessors-in-interest
have been in open, continuous, exclusive, and notorious possession and occupation of the
land in question since 12 June 1945 or prior thereto; that the subject land exceeds the
twelve (12)-hectare limit for confirmation of imperfect title set by Section 47 of
Commonwealth Act (C.A.) No. 141, as amended by Republic Act (R.A.) No. 6940; and
that the subject land forms part of the public domain belonging to the Republic and, thus,
not subject to private appropriation.

RTC granted RRDC's application for registration of the subject land. It opined that the
CENRO certification, stating that the subject land is alienable and disposable and not
covered by any public land application, is sufficient to show the character of the land. It
further ruled, that RRDC and its predecessors-in-interest had been in open and
continuous possession under a bona fide claim of ownership over the subject land based
on the documentary and testimonial evidence offered by RRDC, without discussing how
these pieces of evidence established the required possession. The trial court further
brushed aside the opposition interposed by the heirs of Paulino Avanceña.

CA affirmed RTC. With regards to 12-hectare limit, CA that it was not violated. It
explained that Section 3 of Article XII of the 1987 Constitution, the constitutional
provision which provided for the 12-hectare limit in the acquisition of land, covers only
agricultural lands of the public domain. are limit in the acquisition of land, covers only
agricultural lands of the public domain. It ratiocinated that when the subject land
was acquired through acquisitive prescription by RRDC's predecessors-in interest, it
was converted into a private property and, as such, it ceased to be part of the public
domain. Hence, this petition.

ISSUES:
A. Whether Section 3, Article XII of the 1987 Constitution applies to private lands.
B. Whether RRDC sufficiently established its compliance with the provisions of either
a) Section 14(1) or;
b) Section 14(2) of P.D. No. 1529.

SC: Petition is meritorious. (Granted)

A.
Section 3, Article XII applies only to lands of the public domain. Private lands are,
therefore, outside of the prohibitions and limitations stated therein. Thus, the appellate
court correctly declared that the 12-hectare limitation on the acquisition of lands under
Section 3, Article XII of the 1987 Constitution has no application to private lands.

The limitation and the prohibition on corporations to acquire lands, do not cover
ownership of private lands. Stated differently, whether RRDC can acquire the subject
land and to what extent, depends on whether the pieces of evidence it presented before
the trial court sufficiently established that the subject land is alienable and disposable
land of the public domain; and that the nature and duration of the possession of its
individual predecessors-in-interest converted the subject land to private land by operation
of law.

B.
RRDC failed to establish compliance with the requirements for registration for both
Section 14(1) and Section 14(2).
Rep. vs. CA and Naguit, G.R. No.144057, Jan. 17, 2005

Naguit filed a petition for registration of title through judicial confirmation of


respondent’s imperfect title over a parcel of land. The evidence on record reveals that the
subject parcel of land was originally declared for taxation purposes in the name of Ramon
Urbano (Urbano) in 1945.

Naguit, and her predecessors were in open, continuous and exclusive possession in the
concept of an owner without any objection from any private person or even the
government.

The OSG filed a motion for reconsideration on the ground that the property which is in
open, continuous and exclusive possession must first be alienable. Naguit could not have
maintained a bona fide claim of ownership since the subject lot was declared A and D
only in October 15, 1980. The alienable and disposable character of the land should have
already been established since June 12, 1945 or earlier.

Doctrine: Section 14(1) of the Property Registration Decree merely requires the property
sought to be registered as already alienable and disposable at the time the application for
registration of title is filed.

The more reasonable interpretation of Section 14(1) is that it merely requires the property
sought to be registered as already alienable and disposable at the time the application for
registration of title is filed. If the State, at the time the application is made, has not yet
deemed it proper to release the property for alienation or disposition, the presumption is
that the government is still reserving the right to utilize the property; hence, the need to
preserve its ownership in the State irrespective of the length of adverse possession even if
in good faith.

However, if the property has already been classified as alienable and disposable, as it is
in this case, then there is already an intention on the part of the State to abdicate its
exclusive prerogative over the property
Republic of the Philippines v Jeremias and David Herbieto, G.R. No. 156117

Respondents in the present Petition are the Herbieto brothers, Jeremias and David,
who filed with the MTC, on 23 September 1998, a single application for registration
of two parcels of land, Lots No. 8422 and 8423, located in Cabangahan, Consolacion,
Cebu. They claimed to be the owners in fee simple of the Subjects Lots, which they
purchased from their parents, spouses Gregorio Herbieto and Isabel Owatan, on 25
June 1976.

Petitioner’s Contention: Respondents failed to establish that they and their


predecessors-in-interest had been in open, continuous, and adverse possession of
the Subject Lots in the concept of owners since 12 June 1945 or earlier. According to
the petitioner Republic, possession of the Subject Lots prior to 25 June 1963 cannot
be considered in determining compliance with the periods of possession required
by law. The Subject Lots were classified as alienable and disposable only on 25 June
1963, per CENRO's certification.  It also alleges that the Court of Appeals, in applying
the 30-year acquisitive prescription period, had overlooked the ruling in Republic v.
Doldol, where this Court declared that Commonwealth Act No. 141, otherwise
known as the Public Land Act, as amended and as it is presently phrased, requires
that possession of land of the public domain must be from 12 June 1945 or earlier,
for the same to be acquired through judicial confirmation of imperfect title.

Respondent’s Contention: Alleged that they obtained the title to the Subject Lots
by Purchase from their parents, spouses Gregorio Herbieto and Isabel Owatan, on
25 June 1976. Respondent Jeremias, in his testimony claimed that his parents had
been in possession of the Subject Lots in the concept of an owner since 1950.

MTC: promulgated its Judgment ordering the registration and confirmation of the
title of respondent Jeremias over Lot No. 8422 of the title of respondent Jeremias
over Lot No. 8422 and of respondent David over Lot No. 8423.

CA: The Court of Appeals affirmed the decision of the MTC. The reasoning is that the
land sought to be registered has been classified as within the alienable and
disposable since June 25, 1963. Further, the respondent’s predecessors-in-interest
had occupied and possessed the subject land openly, continuously, exclusively and
adversely since 1950.

SC RULING:

Respondents failed to comply with the required period of possession of the


Subject Lots for the judicial confirmation or legalization of imperfect or
incomplete title.

According to the DENR- CENRO Certification, submitted by respondents themselves,


the Subject Lots are "within Alienable and Disposable, Block I, Project No. 28 per LC
Map No. 2545 of Consolacion, Cebu certified under Forestry Administrative Order
No. 4-1063, dated June 25, 1963. Likewise, it is outside Kotkot-Lusaran Mananga
Watershed Forest Reservation per Presidential Proclamation No. 932 dated June 29,
1992."33 The Subject Lots are thus clearly part of the public domain, classified as
alienable and disposable as of 25 June 1963.

The respondents herein filed their application before the MTC, then it can be
reasonably inferred that they are seeking the judicial confirmation or legalization of
their imperfect or incomplete title over the Subject Lots.

Judicial confirmation or legalization of imperfect or incomplete title to land, not


exceeding 144 hectares, may be availed of by persons identified under Section 48 of
the Public Land Act, as amended by Presidential Decree No. 1073, which reads –

Section 48. The following-described citizens of the Philippines, occupying


lands of the public domain or claiming to own any such lands or an interest
therein, but whose titles have not been perfected or completed, may apply to
the Court of First Instance of the province where the land is located for
confirmation of their claims and the issuance of a certificate of title
thereafter, under the Land Registration Act, to wit:

(a) [Repealed by Presidential Decree No. 1073].

(b) Those who by themselves or through their predecessors-in-


interest have been in open, continuous, exclusive, and notorious
possession and occupation of agricultural lands of the public domain,
under a bona fide claim of acquisition of ownership, since June 12,
1945, or earlier, immediately preceding the filing of the applications
for confirmation of title, except when prevented by war or force
majeure. These shall be conclusively presumed to have performed all
the conditions essential to a Government grant and shall be entitled to
a certificate of title under the provisions of this chapter.

(c) Members of the national cultural minorities who by themselves or


through their predecessors-in-interest have been in open, continuous,
exclusive and notorious possession and occupation of lands of the
public domain suitable to agriculture whether disposable or not,
under a bona fide claim of ownership since June 12, 1945 shall be
entitled to the rights granted in subsection (b) hereof.

Not being members of any national cultural minorities, respondents may only be
entitled to judicial confirmation or legalization of their imperfect or incomplete title
under Section 48(b) of the Public Land Act, as amended. Section 48(b), as amended,
now requires adverse possession of the land since 12 June 1945 or earlier. In the
present Petition, the Subject Lots became alienable and disposable only on 25 June
1963. Any period of possession prior to the date when the Subject Lots were
classified as alienable and disposable is inconsequential and should be excluded
from the computation of the period of possession; such possession can never ripen
into ownership and unless the land had been classified as alienable and disposable,
the rules on confirmation of imperfect title shall not apply thereto. 41 It is very
apparent then that respondents could not have complied with the period of
possession required by Section 48(b) of the Public Land Act, as amended, to acquire
imperfect or incomplete title to the Subject Lots that may be judicially confirmed or
legalized.

WHEREFORE, based on the foregoing, the instant Petition is GRANTED. The


Decision of the Court of Appeals in CA-G.R. CV No. 67625, dated 22 November 2002,
is REVERSED. The Judgment of the MTC of Consolacion, Cebu in LRC Case No. N-75,
dated 21 December 1999, and its Order, dated 02 February 2000 are declared NULL
AND VOID. Respondents' application for registration is DISMISSED.
HEIRS OF MALABANAN V REPUBLIC OF THE PHILIPPINES, G.R. NO. 179987

The property subject of the application for registration is a parcel of land situated in
Barangay Tibig, Silang Cavite, more particularly identified as Lot 9864-A, Cad-452-D,
with an area of 71,324-square meters. On February 20, 1998, applicant Mario
Malabanan, who had purchased the property from Eduardo Velazco, filed an
application for land registration covering the property in the Regional Trial Court
(RTC) in Tagaytay City, Cavite, claiming that the property formed part of the
alienable and disposable land of the public domain, and that he and his
predecessors-in-interest had been in open, continuous, uninterrupted, public and
adverse possession and occupation of the land for more than 30 years, thereby
entitling him to the judicial confirmation of his title.

To prove that the property was an alienable and disposable land of the public
domain, Malabanan presented during trial a certification dated June 11, 2001 issued
by the Community Environment and Natural Resources Office (CENRO) of the
Department of Environment and Natural Resources (DENR), which reads:

This is to certify that the parcel of land designated as Lot No. 9864 Cad 452-D, Silang
Cadastre as surveyed for Mr. Virgilio Velasco located at Barangay Tibig, Silang,
Cavite containing an area of 249,734 sq. meters as shown and described on the Plan
Ap-04-00952 is verified to be within the Alienable or Disposable land per Land
Classification Map No. 3013 established under Project No. 20-A and approved as
such under FAO 4-1656 on March 15, 1982.

Petitioner’s Contention:  the petitioners submit that the mere classification of the
land as alienable or disposable should be deemed sufficient to convert it into
patrimonial property of the State. Relying on the rulings in Spouses De Ocampo v.
Arlos,7 Menguito v. Republic8 and Republic v. T.A.N. Properties, Inc., 9 they argue that
the reclassification of the land as alienable or disposable opened it to acquisitive
prescription under the Civil Code; that Malabanan had purchased the property from
Eduardo Velazco believing in good faith that Velazco and his predecessors-in-
interest had been the real owners of the land with the right to validly transmit title
and ownership thereof; that consequently, the ten-year period prescribed by Article
1134 of the Civil Code, in relation to Section 14(2) of the Property Registration
Decree, applied in their favor; and that when Malabanan filed the application for
registration on February 20, 1998, he had already been in possession of the land for
almost 16 years reckoned from 1982, the time when the land was declared alienable
and disposable by the State.

RTC: Granted Malabanan’s application for land registration.

CA: Reversed the decision of RTC and dismissing the application for registration of
Malabanan. Citing the ruling in Republic v. Herbieto (Herbieto), 4 the CA declared
that under Section 14(1) of the Property Registration Decree, any period of
possession prior to the classification of the land as alienable and disposable was
inconsequential and should be excluded from the computation of the period of
possession. Noting that the CENRO-DENR certification stated that the property had
been declared alienable and disposable only on March 15, 1982, Velazco’s
possession prior to March 15, 1982 could not be tacked for purposes of computing
Malabanan’s period of possession.

SC RULING:
We deny the motions for reconsideration.

The core of the controversy herein lies in the proper interpretation of Section 11(4),
in relation to Section 48(b) of the Public Land Act, which expressly requires
possession by a Filipino citizen of the land since June 12, 1945, or earlier, viz:

Section 48. The following-described citizens of the Philippines, occupying lands of


the public domain or claiming to own any such lands or an interest therein, but
whose titles have not been perfected or completed, may apply to the Court of First
Instance of the province where the land is located for confirmation of their claims
and the issuance of a certificate of title thereafter, under the Land Registration Act,to
wit:

(b) Those who by themselves or through their predecessors-in-interest have been in


open, continuous, exclusive, and notorious possession and occupation of alienable
and disposable lands of the public domain, under a bona fide claim of acquisition of
ownership, since June 12, 1945, or earlier, immediately preceding the filing of the
applications for confirmation of title, except when prevented by war or force
majeure. These shall be conclusively presumed to have performed all the conditions
essential to a Government grant and shall be entitled to a certificate of title under
the provisions of this chapter.

Note that Section 48(b) of the Public Land Act used the words "lands of the public
domain" or "alienable and disposable lands of the public domain" to clearly signify
that lands otherwise classified, i.e., mineral, forest or timber, or national parks, and
lands of patrimonial or private ownership, are outside the coverage of the Public
Land Act. What the law does not include, it excludes. The use of the descriptive
phrase "alienable and disposable" further limits the coverage of Section 48(b) to
only the agricultural lands of the public domain as set forth in Article XII, Section 2
of the 1987 Constitution. Bearing in mind such limitations under the Public Land
Act, the applicant must satisfy the following requirements in order for his
application to come under Section 14(1) of the Property Registration Decree, 28 to
wit:

1. The applicant, by himself or through his predecessor-in-interest, has been


in possession and occupation of the property subject of the application;

2. The possession and occupation must be open, continuous, exclusive, and


notorious;
3. The possession and occupation must be under a bona fide claim of
acquisition of ownership;

4. The possession and occupation must have taken place since June 12, 1945,
or earlier; and

5. The property subject of the application must be an agricultural land of the


public domain.

Further, An examination of Section 48(b) of the Public Land Act indicates that
Congress prescribed no requirement that the land subject of the registration should
have been classified as agricultural since June 12, 1945, or earlier. As such, the
applicant’s imperfect or incomplete title is derived only from possession and
occupation since June 12, 1945, or earlier. This means that the character of the
property subject of the application as alienable and disposable agricultural land of
the public domain determines its eligibility for land registration, not the ownership
or title over it.
The declaration that land is alienable and disposable also serves to determine the
point at which prescription may run against the State. The imperfect or incomplete
title being confirmed under Section 48(b) of the Public Land Act is title that is
acquired by reason of the applicant’s possession and occupation of the alienable and
disposable agricultural land of the public domain. Where all the necessary
requirements for a grant by the Government are complied with through actual
physical, open, continuous, exclusive and public possession of an alienable and
disposable land of the public domain, the possessor is deemed to have acquired by
operation of law not only a right to a grant, but a grant by the Government, because
it is not necessary that a certificate of title be issued in order that such a grant be
sanctioned by the courts
To reiterate, then, the petitioners failed to present sufficient evidence to establish
that they and their predecessors-in-interest had been in possession of the land since
June 12, 1945. Without satisfying the requisite character and period of possession -
possession and occupation that is open, continuous, exclusive, and notorious since
June 12, 1945, or earlier - the land cannot be considered ipso jure converted to
private property even upon the subsequent declaration of it as alienable and
disposable. Prescription never began to run against the State, such that the land has
remained ineligible for registration under Section 14(1) of the Property Registration
Decree. Likewise, the land continues to be ineligible for land registration under
Section 14(2) of the Property Registration Decree unless Congress enacts a law or
the President issues a proclamation declaring the land as no longer intended for
public service or for the development of the national wealth.1âwphi1

WHEREFORE, the Court DENIES the petitioners' Motion for Reconsideration and
the respondent's Partial Motion for Reconsideration for their lack of merit.
REPUBLIC V BACAS, G.R. NO. 182913

In 1938, Commonwealth President Manuel Luis Quezon (Pres. Quezon) issued


Presidential Proclamation No. 265, which took effect on March 31, 1938, reserving
for the use of the Philippine Army three (3) parcels of the public domain situated in
the barrios of Bulua and Carmen, then Municipality of Cagayan, Misamis Oriental.
The parcels of land were withdrawn from sale or settlement and reserved for
military purposes, "subject to private rights, if any there be."

Land Registration Case No. N-275


The Bacases filed their Application for Registration 3 on November 12, 1964 covering
a parcel of land, together with all the improvements found thereon, located in Patag,
Cagayan de Oro City. They alleged ownership in fee simple of the property and
indicated in their application the names and addresses of the adjoining owners, as
well as a statement that the Philippine Army (Fourth Military Area) recently
occupied a portion of the land by their mere tolerance. On April 10, 1968, based on
the evidence presented by the Bacases, the Land Registration Court (LRC) rendered
a decision8 holding that the applicants had conclusively established their ownership
in fee simple over the subject land and that their possession, including that of their
predecessor-in-interest, had been open, adverse, peaceful, uninterrupted, and in
concept of owners for more than forty (40) years.

Land Registration Case No. N-521


The Chabons filed their Application for Registration 9 on May 8, 1974 covering a
parcel of land located in Carmen-District, Cagayan de Oro City, known as Lot 4357,
Cagayan Cadastre. They alleged ownership in fee simple over the property and
indicated therein the names and addresses of the adjoining owners, but no mention
was made with respect to the occupation, if any, by the Philippine Army. On
February 18, 1976, there being no opposition made, even from the government,
hearing on the application ensued. The LRC then rendered a decision 12 holding that
Chabons’ evidence established their ownership in fee simple over the subject
property and that their possession, including that of their predecessor-in-interest,
had been actual, open, public, peaceful, adverse, continuous, and in concept of
owners for more than thirty (30) years.

PETITIONER’S CONTENTION: With respect to the first argument, the Republic cites
Section 15 of P.D. No. 1529, which requires that applicants for land registration
must disclose the names of the occupants of the land and the names and addresses
of the owners of the adjoining properties. The respondents did not comply with that
requirement which was mandatory and jurisdictional. Citing Pinza v. Aldovino, 43 it
asserts that the LRC had no jurisdiction to take cognizance of the case. Moreover,
such omission constituted fraud or willful misrepresentation. The respondents
cannot invoke the indefeasibility of the titles issued since a "grant tainted with fraud
and secured through misrepresentation is null and void and of no effect
whatsoever."44
On the second argument, the Republic points out that Presidential Proclamation No.
265 reserved for the use of the Philippine Army certain parcels of land which
included Lot No. 4354 and Lot No. 4357. Both lots were, however, allowed to be
registered. Lot No. 4354 was registered as OCT No. 0-0358 and Lot No. 4357 as OCT
No. O-669.

The Republic asserts that being part of the military reservation, these lots are
inalienable and cannot be the subject of private ownership. Being so, the
respondents do not have registrable rights over them. Their possession of the land,
however long, could not ripen into ownership, and they have not shown proof that
they were entitled to the land before the proclamation or that the said lots were
segregated and withdrawn as part thereof.

RESPONDENT’S CONTENTION (BACASES): the Bacases stress that there was no


extrinsic fraud because their application substantially complied with the
requirements when they indicated that Camp Evangelista was an occupant by mere
tolerance of Lot No. 4354. On jurisdiction, the Bacases assert that even in the case of
Republic v. Estonilo,47 it was recognized in Presidential Proclamation No. 265 that
the reservation was subject to private rights. They cited the RTC statement that "the
parcels of land they applied for in those registration proceedings and for which
certificates of title were issued in their favor are precisely exempted from the
operation and effect of said presidential proclamation when the very same
proclamation in itself made a proviso that the same will not apply to lands with
existing ‘private rights’ therein.

RESPONDENT’S CONTENTION (CHABONS): the Chabons insist that the CA was


correct when it stated that there was substantial compliance 52 with the
requirements of the P.D. No. 1529 because they expressly stated in their application
that Camp Evangelista was occupying a portion of it. It is contrary to reason or
common sense to state that Camp Evangelista is an adjoining owner when it is
occupying a portion thereof. The Chabons also assailed the proclamation because
when it was issued, they were already the private owners of the subject parcels of
land and entitled to protection under the Constitution. The taking of their property
in the guise of a presidential proclamation is not only oppressive and arbitrary but
downright confiscatory.54

RTC: As the facts and issues in both cases were substantially the same and identical,
and the pieces of evidence adduced were applicable to both, the cases were
consolidated and jointly tried. Thereafter, a joint decision dismissing the two
complaints of the Republic was rendered.In dismissing the complaints, the RTC
explained that the stated fact of occupancy by Camp Evangelista over certain
portions of the subject lands in the applications for registration by the respondents
was a substantial compliance with the requirements of the law. 24 It would have been
absurd to state Camp Evangelista as an adjoining owner when it was alleged that it
was an occupant of the land. 25 Thus, the RTC ruled that the respondents did not
commit fraud in filing their applications for registration.
CA: On November 12, 2007, the CA affirmed the ruling of the RTC. It explained that
once a decree of registration was issued under the Torrens system and the
reglementary period had passed within which the decree may be questioned, the
title was perfected and could not be collaterally questioned later on. 35 Even
assuming that an action for the nullification of the original certificate of title may
still be instituted, the review of a decree of registration under Section 38 of Act No.
496 [Section 32 of Presidential Decree (P.D.) No. 1529] would only prosper upon
proof that the registration was procured through actual fraud, 36 which proceeded
from an intentional deception perpetrated through the misrepresentation or the
concealment of a material fact.37

SC RULING:

The application of the Bacases and the Chabons were filed on November 12, 1964
and May 8, 1974, respectively. Accordingly, the law governing the applications was
Commonwealth Act (C.A.) No. 141, 63 as amended by RA 1942,64 particularly Sec.
48(b) which provided that:

Those who by themselves or through their predecessors in interest have been in


open, continuous, exclusive and notorious possession and occupation of agricultural
lands of the public domain, under a bona fide claim of acquisition of ownership, for
at least thirty years immediately preceding the filing of the application for
confirmation of title except when prevented by war or force majeure. These shall be
conclusively presumed to have performed all the conditions essential to a
Government grant and shall be entitled to a certificate of title under the provisions
of this chapter.

Such power of the President to segregate lands was provided for in Section 64(e) of
the old Revised Administrative Code and C.A. No. 141 or the Public Land Act. Later,
the power of the President was restated in Section 14, Chapter 4, Book III of the
1987 Administrative Code. When a property is officially declared a military
reservation, it becomes inalienable and outside the commerce of man. 66 It may not
be the subject of a contract or of a compromise agreement. 67 A property continues to
be part of the public domain, not available for private appropriation or ownership,
until there is a formal declaration on the part of the government to withdraw it from
being such

The claims of persons who have settled on, occupied, and improved a parcel of
public land which is later included in a reservation are considered worthy of
protection and are usually respected, but where the President, as authorized by law,
issues a proclamation reserving certain lands and warning all persons to depart
therefrom, this terminates any rights previously acquired in such lands by a person
who was settled thereon in order to obtain a preferential right of purchase. And
patents for lands which have been previously granted, reserved from sale, or
appropriate, are void.
Regarding the subject lots, there was a reservation respecting "private rights." In
Republic v. Estonilo,70 where the Court earlier declared that Lot No. 4318 was part
of the Camp Evangelista Military Reservation and, therefore, not registrable, it noted
the proviso in Presidential Proclamation No. 265 requiring the reservation to be
subject to private rights as meaning that persons claiming rights over the reserved
land were not precluded from proving their claims. Stated differently, the said
proviso did not preclude the LRC from determining whether or not the respondents
indeed had registrable rights over the property.
As there has been no showing that the subject parcels of land had been segregated
from the military reservation, the respondents had to prove that the subject
properties were alienable and disposable land of the public domain prior to its
withdrawal from sale and settlement and reservation for military purposes under
Presidential Proclamation No. 265.

In this case, however, the respondents miserably failed to prove that, before the
proclamation, the subject lands were already private lands. They merely relied on
such "recognition" of possible private rights. In their application, they alleged that at
the time of their application,71 they had been in open, continuous, exclusive, and
notorious possession of the subject parcels of land for at least thirty (30) years and
became its owners by prescription. There was, however, no allegation or showing
that the government had earlier declared it open for sale or settlement, or that it
was already pronounced as inalienable and disposable.

It is well-settled that land of the public domain is not ipso facto converted into a
patrimonial or private property by the mere possession and occupation by an
individual over a long period of time.

WHEREFORE, the petition is GRANTED. The November 12, 2007 Decision and the
May 15, 2008 Resolution of the Court of Appeals in CAG.R. CV No. 64142 are
hereby REVERSED and SET ASIDE. Judgment is rendered declaring the proceedings
in the Land Registration Court as NULL and VOID for lack of jurisdiction.
Accordingly, Original Certificate of Title Nos. 0-358 and 0-669 issued by the Registry
of Deeds of Cagayan de Oro City are CANCELLED. Lot No. 4354 and Lot No. 4357 are
ordered reverted to the public domain.
SPS. FORTUNA V REPUBLIC, G.R. NO. 173423.

In December 1994, the spouses Fortuna filed an application for registration of a


2,597-square meter land identified as Lot No. 4457, situated in Bo. Canaoay, San
Fernando, La Union. The application was filed with the RTC and docketed as LRC No.
2372.

PETITIONER’S CONTENTION: stated that Lot No. 4457 was originally owned by
Pastora Vendiola, upon whose death was succeeded by her children, Clemente and
Emeteria Nones. Through an affidavit of adjudication dated August 3, 1972,
Emeteria renounced all her interest in Lot No. 4457 in favor of Clemente. Clemente
later sold the lot in favor of Rodolfo Cuenca on May 23, 1975. Rodolfo sold the same
lot to the spouses Fortuna through a deed of absolute sale dated May 4, 1984. The
spouses Fortuna claimed that they, through themselves and their predecessors-in-
interest, have been in quiet, peaceful, adverse and uninterrupted possession of Lot
No. 4457 for more than 50 years, and submitted as evidence the lot’s survey plan,
technical description, and certificate of assessment. Further, they contend that the
applicable law is Section 48(b) of Commonwealth Act No. 141 or the Public Land Act
(PLA), as amended by Republic Act (RA) No. 1942. RA No. 1942 amended the PLA by
requiring 30 years of open, continuous, exclusive, and notorious possession to
acquire imperfect title over an agricultural land of the public domain. The spouses
Fortuna point out that PD No. 1073 was issued on January 25, 1977 and published
on May 9, 1977; and the PRD was issued on June 11, 1978 and published on January
2, 1979. On the basis of the Court’s ruling in Tañ ada, et al. v. Hon. Tuvera, etc., et
al.,13 they allege that PD No. 1073 and the PRD should be deemed effective only on
May 24, 1977 and January 17, 1979, respectively.

RTC: Granted the application for registration in favor of the spouses Fortuna. The
RTC declared that “[ the spouses Fortuna] have established possession, including
that of their predecessors-in-interest of the land sought to be registered has been
open, continuous, peaceful, adverse against the whole world and in the concept of an
owner since 1948, or for a period of over fifty (50) years.

CA: Reversed and set aside the RTC decision. Although it found that the spouses
Fortuna were able to establish the alienable and disposable nature of the land, they
failed to show that they complied with the length of possession that the law
requires, i.e. since June 12, 1945.

SC RULING:

We deny the petition for failure of the spouses Fortuna to sufficiently prove their
compliance with the requisites for the acquisition of title to alienable lands of the
public domain.

As mentioned, the PLA is the law that governs the grant and disposition of alienable
agricultural lands. Under Section 11 of the PLA, alienable lands of the public domain
may be disposed of, among others, by judicial confirmation of imperfect or
incomplete title. This mode of acquisition of title is governed by Section 48(b) of the
PLA.

Under the PD No. 1073 amendment, possession of at least 32 years – from 1945 up
to its enactment in 1977 – is required. This effectively impairs the vested rights of
applicants who had complied with the 30-year possession required under the RA
No. 1942 amendment, but whose possession commenced only after the cut-off date
of June 12, 1945 was established by the PD No. 1073 amendment. To remedy this,
the Court ruled in Abejaron v. Nabasa that "Filipino citizens who by themselves or
their predecessors-in-interest have been, prior to the effectivity of P.D. 1073 on
January 25, 1977, in open, continuous, exclusive and notorious possession and
occupation of agricultural lands of the public domain, under a bona fide claim of
acquisition of ownership, for at least 30 years, or at least since January 24, 1947
may apply for judicial confirmation of their imperfect or incomplete title under Sec.
48(b) of the [PLA]." January 24, 1947 was considered as the cut-off date as this was
exactly 30 years counted backward from January 25, 1977 – the effectivity date of
PD No. 1073. Accordingly, Section 6 of PD No. 1073 should be understood to mean
that the decree took effect only upon its publication, or on May 9, 1977. This,
therefore, moves the cut-off date for applications for judicial confirmation of
imperfect or incomplete title under Section 48(b) of the PLA to May 8, 1947. In other
words, applicants must prove that they have been in open, continuous, exclusive and
notorious possession and occupation of agricultural lands of the public domain,
under a bona fide claim of acquisition of ownership, for at least 30 years, or at least
since May 8, 1947.

The spouses Fortuna were unable to prove


that they possessed Lot No. 4457 since May 8, 1947

Even if the Court assumes that Lot No. 4457 is an alienable and disposable
agricultural land of the public domain, the spouses Fortuna’s application for
registration of title would still not prosper for failure to sufficiently prove that they
possessed the land since May 8, 1947.

The spouses Fortuna’s allegation that: (1) the absence of a notation that Tax
Declaration No. 8366 was a new tax declaration and (2) the notation stating that Tax
Declaration No. 8366 cancels the earlier Tax Declaration No. 10543 both indicate
that Pastora possessed the land prior to 1948 or, at the earliest, in 1947. We also
observe that Tax Declaration No. 8366 contains a sworn statement of the owner that
was subscribed on October 23, 1947. 34 While these circumstances may indeed
indicate possession as of 1947, none proves that it commenced as of the cut-off date
of May 8, 1947. Even if the tax declaration indicates possession since 1947, it does
not show the nature of Pastora’s possession. Notably, Section 48(b) of the PLA
speaks of possession and occupation. "Since these words are separated by the
conjunction and, the clear intention of the law is not to make one synonymous with
the other. Possession is broader than occupation because it includes constructive
possession. When, therefore, the law adds the word occupation, it seeks to delimit
the all encompassing effect of constructive possession. Taken together with the
words open, continuous, exclusive and notorious, the word occupation serves to
highlight the fact that for an applicant to qualify, his possession must not be a mere
fiction."35 Nothing in Tax Declaration No. 8366 shows that Pastora exercised acts of
possession and occupation such as cultivation of or fencing off the land. Indeed, the
lot was described as "cogonal."

Where, as in this case, no such proof would be forthcoming, there is no justification


for viewing such claim with favor. It is a basic assumption of our polity that lands of
whatever classification belong to the state. Unless alienated in accordance with law,
it retains its rights over the same as do minus.

WHEREFORE, the petition is DENIED. The decision dated May 16, 2005 and the
resolution dated June 27, 2006 of the Court of Appeals in CA-G.R. CV No. 71143 are
AFFIRMED insofar as these dismissed the spouses Antonio and Erlinda Fortuna's
application of registration of title on the basis of the grounds discussed above. Costs
against the spouses Fortuna.
REPUBLIC V GUERERO, G.R. NO. 133168

Sometime in December 1964, respondent Benjamin Guerrero filed with the Bureau
of Lands (now Lands Management Bureau) a Miscellaneous Sales Application No. V-
83191 covering a parcel of land situated at Pugad Lawin, Quezon City, consisting of
256 square meters. Upon favorable report and recommendation of the District Land
Officer, Guerrero’s application was approved per Order of Award (Exhibit "B"), with
the boundaries of the land awarded specified as follows: N-Lot No. 10-C, Psd-37801;
S-Culiat Creek; E-Road; and W-Public Land. A sketch of the land awarded is
contained at the back of the Order of Award.

On July 29, 1983, one Angelina Bustamante filed a protest with the Bureau of Lands
claiming that respondent obtained the sales patent through fraud, false statement of
facts and/or omission of material facts considering that 174 square meters awarded
to respondent covered the land where her house is situated and where she has been
residing since 1961.

Pursuant to the order of the Office of the President, an ocular investigation and
relocation survey was conducted by the DENR. A report (Exhibit "K") was thereafter
submitted with a finding that 83 square meters of the titled property of Guerrero
consisting of 174 square meters is under ACTUAL PHYSICAL POSSESSION of
Marcelo Bustamante (husband of Angelina Bustamante) with only 91 square meters
under the physical possession of Guerrero. It was also found out that OCT No. 0-28
is supposed to be traversed by a road 3 meters wide, as even the Order of Award in
favor of Guerrero, shows by the boundaries of the land indicated therein, viz:
bounded on the N-Lot No. 10-C, Psd-37801, S-Culiat Creek, E-Road and W-Public
Land.

PETITIONER’S CONTENTION: argues that respondent procured his sales patent


and certificate of title through fraud and misrepresentation. To support its basic
posture, petitioner points to the verification survey conducted by Engr. Ernesto
Erive of the DENR, which, to petitioner, argues for the proposition that respondent’s
entitlement to a public land award should have been limited to a 91- square meter
area instead of the 174 square meters eventually granted.

RESPONDENT’S CONTENTION: his OCT No. 0-28 which he secured pursuant to a


sales patent is conslusive and indefeasible under the Torrens system of registration.
As such, his title can no longer be altered, impugned or cancelled.

DIRECTOR OF LANDS: Dismissing the the protest of Angelina Z. Bustamante

MINISTER OF NATURAL RESOURCES: AFFIRMED THE DISMISSAL OF PROTEST

RTC: Dismissed the motion to dismiss. The petitioner Republic failed to prove its
allegation that respondent obtained the sales patent and the certificate of title
through fraud and misrepresentation, rendered judgment finding for the latter. The
trial court likewise ruled that the original certificate of title (OCT No. 0-28 in the
name of respondent acquired the characteristics of indefeasibility after the
expiration of one year from the entry of the decree of registration.

CA: Affirmed the decision of trial court. It is a settled rule that a certificate of title
issued pursuant to any grant or patent involving public lands is as conclusive and
indefeasible as any other certificate of title issued upon private lands in ordinary or
cadastral registration proceedings. The effect of registration of a homestead or any
other similar patent and the issuance of a certificate of title to the patentee is to vest
in him an incontestable title to the land, in the same manner as if ownership had
been determined by final decree of the court, and the title so issued is absolutely
conclusive and indisputable.

SC RULING:

Petitioner relies heavily on the verification survey report which stated that


respondent Guerrero was entitled to only 91 square meters of the subject lot instead
of 174 square meters which was awarded to him. There is, however, no proof that
the area eventually awarded to respondent was intentionally and fraudulently
increased. It was never proven that respondent was a party to any fraud that led to
the award of a bigger area of 174 square meters instead of 91 square meters.
Petitioner even failed to give sufficient proof of any error which may have been
committed by its agents who had surveyed the subject property nor had petitioner
offered a sensible explanation as to the reason for such discrepancy. Thus, the
presumption of regularity in the performance of official functions must be
respected.

This Court agrees with the RTC that the issuance of the sales patent over the subject
lot was made in accordance with the procedure laid down by Commonwealth Act
No. 141, as amended, otherwise known as the Public Land Act. Under Section 91
thereof, an investigation should be conducted for the purpose of ascertaining the
veracity of the material facts set out in the application. The law also requires
sufficient notice to the municipality and barrio where the land is located in order to
give adverse claimants the opportunity to present their claims.

If there is any adverse claim to the land, such claim must be filed at the Bureau of
Lands, Manila on or before the date of the sale; otherwise such claim shall forever be
barred.

Further, the "Order of Award"  dated May 20, 1971, as well as the "Issuance of
Patent" dated June 28, 1982 were both duly signed by the Director of Lands. The
"Order of Award" even declared that Guerrero has in good faith established his
residence on the land in question. On the other hand, the "Issuance of Patent" stated
that the land consisting of 174 square meters is free from any adverse claim and
that Guerrero has fully paid the purchase price of the lot. Having complied with all
the requirements of the law preliminary to the issuance of the patent, respondent
was thus issued MSP No. 8991 dated August 16, 1982. Thereafter, the corresponding
OCT No. 0-28 was issued on August 27, 1982 in the name of respondent Guerrero

At any rate, by legal presumption, public officers are deemed to have regularly
performed their official duties. Thus, the proceedings for land registration that led
to the issuance of MSP No. 8991 and OCT No. 0-28 in respondent’s name are
presumptively regular and proper. To overturn this legal presumption will not only
endanger judicial stability, but also violate the underlying principle of the Torrens
system. Indeed, to do so would reduce the vaunted legal indefeasibility of Torrens
titles to meaningless verbiage. Besides, this presumption of regularity has not been
overcome by the evidence presented by petitioner. We, therefore, cannot sustain
petitioner’s contention that fraud tainted the sales patent granted to respondent
Guerrero, as well as the certificate of title issued in consequence thereof.

Granting that Guerrero committed extrinsic and actual fraud, petitioner failed to
avail itself of the remedy within the prescribed period. Under Section 38 of Act No.
496, a petition for reopening and review of the decree of registration must be filed
within one year from the date of entry of said decree.

In the case of public land grants or patents, the one-year period commences from
the issuance of the patent by the government.

In the instant case, the sales patent was issued to respondent on August 16, 1982,
while petitioner instituted an action to amend respondent’s certificate of title on
November 7, 1989 or after the lapse of more than seven (7) years from the issuance
of the patent. Clearly, petitioner failed to timely avail of the remedy to contest
Guerrero’s title.

WHEREFORE, the instant petition is hereby DENIED and the assailed decision is
AFFIRMED.
DREAM VILLAGE V. BASES DEVELOPMENT AUTHORITY G. R. No. 192896

FACTS:

Petitioner Dream Village Neighborhood Association, Inc. (Dream Village) claims to


represent more than 2,000 families who have been occupying a 78,466-square
meter lot in Western Bicutan, Taguig City since 1985 "in the concept of owners
continuously, exclusively and notoriously." On October 16, 1987, President Corazon
C. Aquino issued Proclamation No. 172 amending Proclamation No. 2476 by limiting
to Lots 1 and 2 of the survey Swo-13-000298 the areas in Western Bicutan open for
disposition.

Now charging the Bases Conversion and Development Authority (BCDA) of


wrongfully asserting title to Dream Village and unlawfully subjecting its members to
summary demolition, resulting in unrest and tensions among the residents, on
November 22, 1999, the latter fifiled a letter-complaint with the COSLAP to seek its
assistance in the verifification survey of the subject 78,466-sq m property, which
they claimed is within Lot 1 of Swo-13-000298 and thus is covered by Proclamation
No. 172. They claim that they have been occupying the area for thirty (30) years "in
the concept of owners continuously, exclusively and notoriously for several years,"
and have built their houses of sturdy materials thereon and introduced paved roads,
drainage and recreational and religious facilities. Dream Village, thus, asserts that
the lot is not among those transferred to the BCDA under R.A. No. 7227, and
therefore patent applications by the occupants should be processed by the Land
Management Bureau (LMB).

On April 1, 2004, the COSLAP received the fifinal report of the verifification survey
and a blueprint copy of the survey plan from Atty. Rizaldy Barcelo, Regional
Technical Director for Lands of DENR. Specififically, Item No. 3 of the DENR report
states:

3. Lot-1, Swo-000298 is inside Proclamation 172. Dream Village Neighborhood


Association, Inc. is outside Lot-1, Swo-13-000298 and inside Lot-10, 11 & Portion of
Lot 13, Swo-00-0001302 with an actual area of 78,466 square meters. Likewise, the
area actually is outside Swo-00-0001302 of BCDA. On the basis of the DENR’s
verifification survey report, the COSLAP resolved that Dream Village lies outside of
BCDA, and particularly, outside of Swo-00-0001302, and thus directed the LMB of
the DENR to process the applications of Dream Village’s members for sales patent,
noting that in view of the length of time that they "have been openly, continuously
and notoriously occupying the subject property in the concept of an owner, x x x
they are qualifified to apply for sales patent on their respective occupied lots
pursuant to R.A. Nos. 274 and 730 in relation to the provisions of the Public Land
Act."

The CA in its Decision dated September 10, 2009 ruled that the COSLAP has no
jurisdiction over the complaint because the question of whether Dream Village is
within the areas declared as available for disposition in Proclamation No. 172 is
beyond its competence to determine, even as the land in dispute has been under a
private title since 1906, and presently its title is held bya government agency, the
BCDA, in contrast to the case of Bañ aga relied upon by Dream Village, where the
disputed land was part of the public domain and the disputants were applicants for
sales patent thereto. Dream Village’s motion for reconsideration was denied in the
appellate court’s Order of July 13, 2010, thus the petition for Review in the Supreme
Court.

ISSUE:
Whether or not the area occupied by Dream Village, on the basis of the DENR’s
verifification survey report, that sits on the abandoned C-5 Road, which lies outside
the area of BCDA, declared in Proclamation Nos. 2476 and 172 as alienable and
disposable.

HELD:
NO. The petition is DENIED.

The mere fact that the original plan for C-5 Road to cross Swo-00-0001302 was
abandoned by deviating it northward to traverse the southern part of Libingan ng
mga Bayani does not signify abandonment by the government of the bypassed lots,
nor that these lots would then become alienable and disposable. They remain under
the title of the BCDA, even as it is signifificant that under Section 8(d) of R.A. No.
7227, a relocation site of 30.5 has. was to be reserved for families affected by the
construction of C-5 Road. It is nowhere claimed that Lots 10, 11 and 13 of Swo-00-
0001302 are part of the said relocation site. These lots border C-5 Road in the south,
making them commercially valuable to BCDA, a farther argument against a claim
that the government has abandoned them to Dream Village. Article 1113 of the Civil
Code provides that "property of the State or any of its subdivisions not patrimonial
in character shall not be the object of prescription." Thus, under Article 422 of the
Civil Code, public domain lands become patrimonial property only if there is a
declaration that these are alienable or disposable, together with an express
government manifestation that the property is already patrimonial or no longer
retained for public service or the development of national wealth. Only when the
property has become patrimonial can the prescriptive period for the acquisition of
property of the public dominion begin to run.
HWA PING VS AYALA LAND

Principle: The Rule – that between two conflicting titles, the title registered earlier
prevails – is not absolute.

Exception: If it can be clearly ascertained by the ordinary rules of construction


relating to written documents, that the inclusion of the land in the certificate of title
of prior date is a mistake, the mistake may be rectified by holding the latter of the
two certificates of titles to be conclusive.

03/17/1921 Sps. Diaz submitted to GLRO for approval of the Director of Lands a
survey plan located in Sitiio Kay Monica, Barrio Pugad Lawin, Las Pinas, Rizal
covering Lot 1 Survey Plan (PSU-25909) for 460, 626 sq.m.

05/26/1921 Director of Lands approved.


PSU-25909

10/21/1925 Dominador Mayuga submitted another survey plan covering Lot 3 of


the same land located in Sitio May Kokek, Barrio Almanza, Las Pinas, Rizal PSU-
47035

03/06/1931 Alberto Yaptinchay submitted another survey plan covering Lot 2 and
3 of the same parcel of land
PSU-80886/ SWO-20609
Lot 2- 158, 494 sq.m.
Lot 3- 171, 309 sq.m.

05/09/1950 Yaptinchay was issued OCT


OCT No. 242 covering Lot 2 and 3

05/21/1958 Dominador Mayuga was issued OCT


OCT No. 1609 covering Lot 3 from PSU-47035

05/28/1967 CPJ Corp. bought the properties from Mayuga and Yaptinchay OCT No.
242 (PSU- 80886/SWO-20609) OCT No. 1609 (PSU-47035)

02/16/1968 Diaz filed for original registration w/ CFI Pasay on Lot 1 pursuant to
his survey plan

05/21/1970 Subsequently got a favorable judgment from CFI and was issued an
OCT PSU-25909 OCT No. 8510

05/17/1971 DIAZ CASE: CPJ Corp. filed a suit against Diaz to review OCT No. 8510
alleging that the sps committed fraud because interested persons were not notified
of their application for registration OCT No. 8510
08/30/1976 Alberto Diaz sold to Librado Cabautan parcels of land which were
sourced from his OCT No. 8510

05/04/1980 ALI acquired properties from CPJ Corp.


Lot 3; OCT 1609; PSU- 47035
Lot 2; OCT 242; PSU- 80886/SWO-20609
Lot 3; OCT 242; PSU- 80886/SWO-20609
Lot 6; OCT 242; PSU- 80886/SWO-20609

1994 Cabautan sold properties to Sps. Yu from the land he bought from Diaz
Lot 1-B (135,000 sq.m.) and
Lot 1-A (half only with 67, 813 sq.m.)

08/1995 Sps Yu visited their lots and were shocked to find that the lot was already
fenced by ALI and they were prevented from occupying. Sps. Yu’s TCTs overlapped
with properties of ALI

12/31/1995 Diaz case: RTC rendered decision against the Diaz


-OCT 8510 is to be cancelled
-Sps. Diaz appealed to CA

12/4/1996
Yu Case: Sps Yu filed a complaint in RTC Las Pinas against ALI. They declared that
ALI’s titles should be declared null and void because it overlapped with their
property and that the Sps Yu’s predecessors-in-interest can trace their titles back to
1921 (Diaz) then to 1994 (Cabatuan sold properties to them) as well as judicial
confirmation of title Sps Yu alleged that PSU- 47035 and PSU- 80886/SWO-20609
were copied from PSU-25909

RTC Las Pinas ruled in favor of Sps Yu


ALI appealed to CA PSU-47035 and PSU-80886/SWO-20609 had discrepancies. PSU-
25909 had no irregularity. Titles of ALI were void ab initio because they were
secured through fraudulent surveys

Diaz and Yu case was consolidated


06/19/2003 CA: ruled in favor of ALI

Diaz case: Sps Diaz committed fraud, cancel OCT No. 8510 and the TCTs and that
came from it because the Sps Diaz allegedly knew of CPJ Corp.’s interest over the
property yet they failed to stipulate it upon their application for registration

Yu Case: Sps YU can no longer assert the titles because 1 year period to contest the
title had prescribed therefore because ALI possessed an earlier title, their title was
incontestable As the original titles of ALI predated that of Sps Yu, the CA concluded
that the former titles were superior.
A/N: this decision followed the General Rule on superiority of earlier titles

ALI’s title from OCT 242 and 1609 were issued to their predecessors-in-interest on
1950 and 1958 respectively while OCT 8510 was issued only on 1970

Sps Yu and Diaz filed for a motion for reconsideration. CA granted.

02/08/2005 CA: favored Sps Yu and Sps Diaz

Yu Case: PSU-47035 and PSU-80886/SWO- 20609 were seriously erroneous. PSU-


25909 bore all the hallmarks of verity (Registration done fraudulently is no
registration at all prevails over the rules on equity)PSU-80886/SWO-20609 had
defects: doubtful signature of A.N. Feliciano (conducted the survey for Diaz, Mayuga,
Guico and Yaptinchay) and there was even a memorandum from the DENR Regional
Director of Operations not to issue copies of technical descriptions of the said land

Diaz case: Sps Diaz had no obligation to inform CPJ Corp. and its successors about
their registration because the original titles of the latter were based on fraudulent
surveys

ALI filed a motion for reconsideration.

CA granted ALI’s second MOR and reversed and set aside its 2005 Decision
and reverted to its 2003 decision.

The discrepancies in PSU-47035 and PSU-80886/SWO-20609 were immaterial to


assail the validity of OCT Nos. 242, 244 and 1609 which were registered earlier than
OCT 8510.

SC:

in favor of Sps Yu and Diaz


The Court cannot close its eyes to the blatant defects on the surveys upon which the
original titles of ALI were derived simply because its titles were registered. To allow
these certificates of title in the registration books, even though these were sourced
from invalid surveys, would tarnish and damage the Torrens system of registration,
rather than uphold its integrity The Rule – that between two conflicting titles, the
title registered earlier prevails – is not absolute. Merely relying on the date of
registration of the original titles is insufficient because it is the surveys therein that
are being assailed. Exception: If it can be clearly ascertained by the ordinary rules of
construction relating to written documents, that the inclusion of the land in the
certificate of title of prior date is a mistake, the mistake may be rectified by holding
the latter of the two certificates of titles to be conclusive.

Although a certificate of title serves as evidence of an indefeasible and


incontrovertible title to the property in favor of the person whose name appears
therein, it is not a conclusive proof of ownership. Hence, the Court may inquire into
the validity of the ownership of a property by scrutinizing the movant's evidence of
title and the basis of such title. When there is compelling proof that there is doubt on
the validity of the sources or basis of such title, then an examination is proper. Thus,
the surveys of the certificates of title are not immune from judicial scrutiny, in light
of the genuine and legitimate reasons for its analysis.

In fine, the Court finds that there are numerous defects in Psu-47909, Psu-80886
and Psu-80886/SWO-20609, which are all hallmarks of fraud, (to name a few).:

1. That A.N. Feliciano conducted all the surveys even though he should have known
that the earlier dated survey Psu-25909, already covered the same parcel of land;

2. That Psu-47909, Psu-80886 and Psu-25909 covered the same parcel of land and
were conducted by the same surveyor but each survey stated a different location;

3. That the photocopy of Psu-47035, as submitted by ALI, shows that it was done
fora certain Estanislao Mayuga but the certified true copy of Psu- 47035 depicted
that it was made for Dominador Mayuga;

4. That Psu-80886 did not contain the signature of then Director of Lands, Serafin P.
Hidalgo, and it is well-settled rule that no plan or survey may be admitted in land
registration proceedings until approved by the Director of Lands;

5. That the total area of the property covered by Psu-80886 contained many
erasures, which were not satisfactorily explained;

6. That there was a difference in the intensity of the lower right portion of Psu-
80886 which showed that it may simply have been an attachment to the main
document

Thus, the Court holds that plan PSU-25909 is a true copy of an official
document on file with the Bureau of Lands and is, therefore, entitled to great
weight and appreciation, there being no irregularity demonstrated in the
preparation thereof.
REPUBLIC V. NICOLAS, G.R. No. 181435, Oct. 2, 2017

FACTS:

1. In 1996, respondent filed a Petition before the RTC of San Mateo, Rizal, seeking to
register her title over Lot 2 of Survey Plan Psu-213331, a parcel of land located in
Brgy. San Isidro, Rodriguez, Rizal, with an area of 118,448 square meters. She
asserted that she was entitled to confirmation and registration of title, as she had
been in "natural, open, public, adverse, continuous, uninterrupted" possession of the
land in the concept of an owner since October 1964.

2. Petitioner Republic of the Philippines contended that

(a) neither respondent nor her predecessors-in interest had been in open,
continuous, exclusive and notorious possession of the land since 12 June 1945;

(b) the Tax Declarations attached to the Petition did not constitute sufficient
evidence of the acquisition or possession
of the property;

(c) respondent failed to apply for registration of title within six months from 16
February 1976 as required by Presidential Decree No. (P.D.) 892; and

(d) the land in question was part of the public domain and not subject to private
appropriation.

3. RTC directed respondent to submit documents to establish that

(a) the property that was the subject of the application for registration of title was
not covered by the Comprehensive Agrarian Reform Program of the Government;

(b) there were no tenants on the property; and

(c) the land was not subject to any homestead, free patent, or grant of title from the
Land Registration Authority (LRA), the Bureau of Lands, or the Department of
Agrarian Reform.

4. The Community Environment and Natural Resources Office (CENRO) submitted a


Report stating that the land "appears to be not covered by any public land
application nor embraced by any administrative title." However, the entry with
respect to whether the land was within the alienable and disposable zone was left
blank with a notation that the area was "not projected due to unavailability of
coordinates re[:] Tala Estate Tie-Line."

5. The LRA likewise submitted a Report stating that it "was not in a position to
verify whether or not the parcel of land subject of registration is already covered by
land patent and is within the area classified as alienable and disposable land of the
public domain." Hence, the LRA recommended that the CENRO of Antipolo, Rizal, be
ordered to submit a report on the status of the land. This proposal was adopted by
the RTC.

6. During trial, respondent presented three witnesses to prove her right to register
the property: Leonila Alfaro, her daughter and attorney-in-fact, who testified that
respondent had occupied the land since 1940 and had paid the real estate taxes
therefor since 1969; Santiago Eulin, who was allegedly hired by respondent to plant
vegetables and fruit trees on the land and who acted as its caretaker since 1942; and
Roberto M. Valdez of the LRA, who identified the original tracing cloth plan for the
property.

7. The following documents were likewise submitted to the trial court: Survey Plan
PSU-213331, a Surveyor's Certificate and technical descriptions of the property,
which purportedly proved that the land had been duly surveyed by the Land
Management Sector; various Tax Declarations and receipts; and a Certification
issued by the CENRO that the land applied for was not covered by any public land
application.

8. RTC – granted the petition and ordered the issuance of a Decree of Registration in
favor of respondent;

9. CA – dismissed petitioner’s appeal; it further characterized the land as private


property

ISSUE:
Whether or not the subject land is a private property.

RULING:
NO.

Applications for registration of title to land, both public and private, are governed by
Section 14 of P.D. 1529:

SECTION 14. Who May Apply. — The following persons may file in the proper Court
of First Instance an application for registration of title to land, whether personally or
through their duly authorized representatives:

(1) Those who by themselves or through their predecessors-in-interest have been in


open, continuous, exclusive and notorious possession and occupation of alienable
and disposable lands of the public domain under a bona fide claim of ownership
since June 12, 1945 or earlier.

(2) Those who have acquired ownership of private lands by prescription under the
provisions of existing laws.
(3) Those who have acquired ownership of private lands or abandoned river beds
by right of accession or accretion under the existing laws.

(4) Those who have acquired ownership of land in any other manner provided for
by law.

Section 14 refers to a distinct type of application depending on the applicable legal


ground. It is important for the Court to first determine the exact legal ground used
by an applicant for registration.

In this case, we note that the application filed by respondent before the RTC did not
state the exact legal basis of her request. At best, the pleading implied that her claim
was one for registration and confirmation of title based on her possession and
occupation of the property.

It appears that the claim of respondent is anchored on either of the first two
paragraphs of Section 14. However, it is unclear whether she sought judicial
confirmation and registration of her title pursuant to Section 14 (1) of P.D. 1529, or
of the registration of her title on the ground of acquisitive prescription under
Section 14 (2) of the same law.

No specific provision in P.D. 1529 was identified by the RTC when it granted the
Petition. Its mention of the Civil Code, however, seems to indicate an application of
the principle of acquisitive prescription. The CA, for its part, delineated the
differences between the first two paragraphs of Section 14, but decided to apply
both clauses. In its Decision, it ruled that respondent is entitled to register her title
under either paragraph.

The Court has examined the application for registration in this case under the legal
framework of both Section 14 (1) and (2) of P.D. 1529. We find that respondent has
failed to sufficiently establish the requisites of both paragraphs; in particular, with
respect to the classification and the character of the land in question. Hence, we are
constrained to reverse the CA and the RTC Decisions allowing the registration of her
title to the property.

Respondent has failed to prove that the property is alienable and disposable
agricultural land that may be registered under Section 14 (1) of P.D. 1529. This
provision grants occupants of public land the right to judicial confirmation of their
title. registration is allowed provided the following requisites have been complied
with:
(1) The applicant is a Filipino citizen.

(2) The applicant, by himself or through his predecessors-in-interest, has been in


open, continuous, exclusive and notorious possession and occupation of the
property since 12 June 1945.
(3) The property has been declared alienable and disposable as of the filing of
the application.

(4) If the area applied for does not exceed 12 hectares, the application should be
filed by 31 December 2020.

Respondent failed to establish the third requisite, i.e., that the property subject of
the application is alienable and disposable agricultural land by virtue of a positive
act of the Executive Department.

The conclusions of the RTC and the CA are not only contradicted by the evidence on
record; they are likewise contrary to law and jurisprudence. As a result, the Court is
constrained to set aside these pronouncements.

To prove that the property subject of an application for original registration is part
of the alienable and disposable lands of the public domain, applicants must
"identify a positive act of the government, such as an official proclamation,
declassifying inalienable public land into disposable land for agricultural or
other purposes."

To sufficiently establish this positive act, they must submit

(1) a certification from the CENRO or the Provincial Environment and Natural
Resources Office (PENRO); and

(2) a copy of the original classification approved by the DENR Secretary and
certified as a true copy by the legal custodian of the official records.

Respondent not only neglected to submit the required CENRO/PENRO certification


and DENR classification, but also presented evidence that completely failed to prove
her assertion.

None of the documents submitted by respondent to the trial court indicated that the
subject property was agricultural or part of the alienable and disposable lands of the
public domain. At most, the CENRO Report and Certification stated that the land was
not covered by any kind of public land application. This was far from an adequate
proof of the classification of the land.

The RTC did not have sufficient basis for its finding that the property in question
was alienable and disposable.

The Court also finds that the ruling of the CA on the evidentiary value of the private
survey is untenable. The fact that the land has been privately surveyed is not
sufficient to prove its classification or alienable character. While the conduct of a
survey and the submission of the original tracing cloth plan are mandatory
requirements for applications for original registration of land under P.D. 1529, they
only serve to establish the true identity of the land and to ensure that the property
does not overlap with another one covered by a previous registration. These
documents do not, by themselves, prove alienability and disposability of the
property.

It was grave error for the CA to consider the mere conduct of a private survey as
proof of the classification and the alienability of the land.

Respondent has failed to prove that the land subject of the application is part
of the patrimonial property of the State that may be acquired by prescription
under Section 14 (2) of P.D. 1529.

The Court finds no sufficient basis to allow the registration of the property under
Section 14 (2).

By express provision of the law, only private lands that have been acquired by
prescription under existing laws may be the subject of applications for registration
under Section 14 (2). The starting point of the Court's evaluation must, therefore, be
whether the property involved falls within the scope of the paragraph.

Under the Civil Code, all things within human commerce are generally susceptible of
prescription.

Properties of the public dominion, or those owned by the State, are expressly
excluded by law from this general rule, unless they are proven to be patrimonial in
character.

To establish that the land subject of the application has been converted into
patrimonial property of the State, an applicant must prove the following:

1. The subject property has been classified as agricultural land.

2. The property has been declared alienable and disposable.

3. There is an express government manifestation that the property is already


patrimonial, or is no longer retained for public service or the development of
national wealth.

It must be emphasized that without the concurrence of these three conditions, the
land remains part of public dominion and thus incapable of acquisition by
prescription.

Respondent has failed to allege or prove that the subject land belongs to the
patrimonial property
of the State.
The DENR-CENRO Certifications submitted by respondent are not enough; they
cannot substitute for the three conditions required by law as proof that the land
may be the subject of prescription under the Civil Code. For the same reason, the
mere conduct of a private survey of a property — even with the approval of the
Bureau of Lands — does not convert the lot into private land or patrimonial
property of the State. Clearly, the appellate court erred when it relied on the survey
to justify its conclusion that the land is private in nature.

Considering the absence of sufficient evidence that the subject land is a patrimonial
property of the State, we must consider it part of public dominion and thus immune
from acquisitive prescription.

As a final note, the Court must point out that proof of the classification, alienability
and disposability of the subject property is of particular significance in applications
for the registration of land. Given the general rule that public lands may not be
alienated, it is the burden of applicants to prove that the land they seek to
register falls within the classifications enumerated in Section 14 of P.D. 1529;
in particular, the specific paragraph they invoke as basis for registration. Absent
that proof, no length of possession or occupation would vest any right of ownership
over the property, and registration under P.D. 1529 cannot be sanctioned by this
Court.

WHEREFORE, the Petition is hereby GRANTED. The Court of Appeals Decision dated
23 August 2007 and Resolution dated 22 January 2008 are REVERSED and SET
ASIDE. Respondent's application for land Registration is DENIED for lack of merit.
CARPO V. AYALA LAND

FACTS:

On February 16, 1995, petitioner spouses Morris and Socorro Carpo (Carpos) filed a
Complaint for Quieting of Title with the RTC of Makati City against Ayala
Corporation, Ayala Property Ventures Corporation (APVC), and the Register of
Deeds of Las Piñ as.

The complaint prayed that the trial court render judgment:


- canceling and declaring void TCT Nos. 125945, T-4366, T-4367, T-4368 and all
alleged derivatives thereof, issued in the name of Ayala Corporation and/or APVC
over the properties or portion thereof embraced in the Carpos’ TCT No. 296463 and
issuing a writ of possession in favor of the Carpos and/or ordering Ayala
Corporation and APVC to surrender to the Carpos the properties or portion thereof
being occupied by the said corporations under inherently invalid or void titles;

- declaring TCT No. 296463 issued in their names as valid and the Carpos as the
owners of the property
described therein "including the parcels of land being claimed and occupied by
Ayala [Corporation] and APVC withou[t] valid and enforceable titles";

- ordering Ayala Corporation and APVC to pay jointly and severally the amount of
₱100,000 as attorney’s fees plus costs of suit and litigation expenses.
Before defendants could file an answer, petitioners filed an Amended Complaint,
impleading respondent Ayala Land, Incorporated (ALI) in lieu of Ayala Corporation
after purportedly verifying with the Register of Deeds of Las Piñ as that the title to
the subject property was registered in the name of ALI and not Ayala Corporation.
In its Amended Answer, ALI alleged that:

- the areas covered by TCT Nos. T-4366, T-4367, and T-4368 do not overlap with the
Carpos’ claimed property and the dispute pertained only to the land covered by the
Carpos’ TCT No. 296463 and TCT No. T-5333 in the name of Las Piñ as Ventures, Inc.
(LPVI) which was derived from TCT No. 125945 in the name of Ayala Corporation.

- Further, ALI alleged that it is the true owner of the property covered by TCT No. T-
5333 as it traces back its title to Original Certificate of Title (OCT) No. 242 issued in
1950 while the Carpos’ title was derived from OCT No. 8575 issued only in 1970.

- ALI also claimed the Carpos’ complaint was barred by res judicata in view of the
1941 decision of this Court in Guico v. San Pedro which upheld the ownership of a
certain Eduardo Guico over the subject property as Lot 3, of Psu-80886 over the
claim of a certain Florentino Baltazar who was asserting ownership of the same
under his plan, Psu-56007.
During the pendency of the case, ALI secured a title in its own name, TCT No. T-
41262, over the property previously covered by TCT No. T-5333.

Makati RTC ruled that the present case was an action in rem and directed the
transfer of the case to the RTC of Las Piñ as where the disputed property is located.
The case was thereafter assigned to Branch 255 of the Las Piñ as RTC. On December
17, 1996, ALI filed a Motion for Summary Judgment on the ground that there was
allegedly no genuine issue as to any material fact and the only issue for the court to
resolve was a purely legal one ― which of the two (2) titles should be accorded
priority. According to ALI, the parties were relying on their respective TCTs, and
since ALI admittedly traces its title to OCT No. 242 which was issued more than
twenty (20) years earlier than the Carpos’ predecessor’s title (OCT No. 8575), its
title is, thus, superior. RTC denied ALI’s motion for summary judgment, however it
was overturned by the CA and ordered the RTC to render a summary judgment.
Both parties filed a motion for reconsideration but it was both denied. Both parties,
then, elevated the matter to this Court in separate petitions for review on certiorari
and in separate minute Resolutions, the Court denied both petitions. Both parties’
motions for reconsideration were likewise denied.

RTC:

- found Carpos’ title superior to that of ALI

- The property being claimed by the defendant ALI, allegedly registered under OCT
No. 242, is shown
to have been surveyed under SWO and not bearing the approval of the Director of
the Bureau of Lands. Any title issued emanating from a survey plan without the
approval of the Director of the Bureau Of Lands is tainted with irregularity and
therefore void, as ruled in Republic Cement Corporation vs. Court of Appeals, et al.,
198 SCRA 734.

- Evidently, the SWO survey of the property which defendant ALI claimed to
have been originated from OCT No. 242 had not been approved by the Director
of the Bureau of Lands, but was apparently prepared and approved by the
then Land Registration Commissioner and under the law, the same is void.

CA:

The instant appeal is GRANTED, the assailed Summary Judgment of the RTC Las
Piñ as is hereby REVERSED and SET ASIDE, and a new one is rendered as follows:

- TCT No. 41262, formerly TCT No. T-5333, in the name of defendant-appellant
Ayala Land,
Incorporated is hereby declared to be the VALID title to the subject property;
- TCT No. 296463 issued in the name of plaintiffs-appellees is declared to be NULL
and VOID;

- The concerned Register of Deeds is hereby ORDERED to cancel plaintiffs-appellees’


TCT No. 296463, and any and all titles issued covering the subject property, for
being spurious and void, and of no force and effect. The Carpos filed their motion for
reconsideration but the same was denied by the CA.

ISSUE:
Whether or not the CA erred in declaring that the title of respondent is valid even
without the requisite survey plan approved by the director of lands.

RULING:

No.

After a thorough review of the records, we deny the petition and concur with the CA
that the Summary Judgment rendered by the trial court should be reversed and set
aside.

Since the existence of two titles over the same property, as well as the fact of
overlapping of the technical descriptions of the two titles are admitted in the
pleadings, and substantiated by the supporting documents attached by the
defendant-movant (petitioner herein) to its Motion for Summary Judgment, there is
no genuine issue as to any material fact. If at all, the sole issue is a legal one, to
wit: whose title (as to the conflicting ones) is superior and must be upheld.
This issue may be decided on the basis of the affidavits and supporting documents
submitted by the parties, as well as the applicable law and jurisprudence on the
matter. The same is true with the other defenses raised by the petitioner in its
responsive pleading, to wit: res judicata, prescription and laches —which may
likewise be resolved without going to trial.

The foregoing CA decision became final and executory after the separate petitions
for review filed with this Court by the parties were denied with finality. The parties,
and even the trial court, were bound by the CA’s factual finding therein that the only
lots whose technical descriptions overlap are those covered by the Carpos’ TCT No.
296463 and ALI’s TCT No. T-5333 which later became TCT No. T-41262. There was
simply no basis for the trial court to invalidate all the ALI titles mentioned in the
complaint. The incorrectness of this sweeping invalidation of ALI titles in the
Summary Judgment is even more evident in the case of TCT No. T-4367 (Lot 2, plan
Psu-47035) and TCT No. T-4368 (Lot 3, plan Psu-47035). Petitioners’ claims with
respect to these properties are already barred by res judicata. Petitioners' claims
with respect to these properties are already barred by res judicata. In Realty Sales
Enterprise, Inc. v. Intermediate Appellate Court, petitioner Morris Carpo already
asserted his purported ownership of these two properties based on a TCT with the
same survey plan number (Psu-56007) as TCT No. 296463. However, in Realty, his
claim was discredited by the Court when it held that Realty Sales Enterprise, Inc.
(Realty), ALI's predecessor in interest, is the one with valid title to these properties.
On December 29, 1977, Morris Carpo filed a complaint with the Court of First
Instance of Rizal, Branch XXIII, presided over by Judge Rizalina Bonifacio Vera
(hereafter referred to as Vera Court), for "declaration of nullity of Decree No. N-
63394 and TCT No. 20408." Named defendants were Realty Sales Enterprise, Inc.,
Macondray Farms, Inc. and the Commissioner of Land Registration.

It appears that it was Estanislao Mayuga, father of Dominador Mayuga, predecessor-


in-interest of Realty, who originally filed on June 24, 1927 a registration proceeding
to confirm his title over parcels of land described as Lots 1, 2 and 3, Plan Psu-47035.
(Lots 2 and 3 are the subject of the instant litigation among Carpo, Realty and
QCDFC.) Carpo bought the disputed property from the Baltazars, the original
registered owners, by virtue of a deed executed before Iluminada Figueroa, Notary
Public of Manila dated October 9, 1970. The Baltazars, predecessors-in-interest of
Carpo are heirs of Florentino Baltazar, an oppositor in the original application filed
by Estanislao Mayuga in 1927. As stated earlier, the CFI-Rizal confirmed the title of
Estanislao to Lots 1, 2 and 3 of Plan Psu-47035. As such successors of Florentino,
they could not pretend ignorance of the land registration proceedings over the
disputed parcels of land earlier initiated by Eduardo Guico, Florentino Baltazar and
Estanislao Mayuga, as when as the decisions rendered therein.

Moreover, it is not disputed that the title in the name of Dominador Mayuga, from
whom Realty derived its title, was issued in 1958, or twelve years before the
issuance of the title in the name of the Baltazars in 1970. In this jurisdiction, it is
settled that "(t)he general rule is that in the case of two certificates of title,
purporting to include the same land, the earlier in date prevails xxx. In
successive registrations, where more than one certificate is issued in respect of a
particular estate or interest in land, the person claiming under the prior certificate is
entitled to the estate or interest; and that person is deemed to hold under the prior
certificate who is the holder of, or whose claim is derived directly or indirectly from
the person who was the holder of the earliest certificate issued in respect thereof
xxx."

Petitioners alleged that the CA erred in declaring that the title of respondent is valid
even without the requisite survey plan approved by the Director of the Bureau of
Lands.

Petitioners clearly misunderstood or deliberately misread the CA’s ruling on this


point. It is the CA’s view that the trial court’s pronouncement that OCT No. 242 was
issued without an approved survey plan was unwarranted in view of the
presumption of regularity that said title enjoys. To begin with, a perusal of the
defendant’s answer or amended answer would show that, contrary to the trial
court’s allusions thereto, there is no admission on the part of ALI that OCT No. 242
was issued without a survey plan that was duly approved by the Director of the
Bureau of Lands. There is likewise no evidence on record to support the trial court’s
finding that the survey plan submitted to support the issuance of OCT No. 242 in the
1950 land registration proceedings was approved only by the Land Registration
Commissioner and not by the Director of the Bureau of Lands. It would appear the
trial court came to the conclusion that OCT No. 242 was issued without a duly
approved survey plan simply because the notation "SWO" appeared in the technical
description of the said title which was attached to the answer and due to ALI’s
failure to allege in its pleadings that the survey plan submitted in support of the
issuance of OCT No. 242 was approved by the Director of the Bureau of Lands. The
Court need not emphasize that it is not for ALI to allege in its pleadings, much
less prove, that its predecessor-in-interest complied with the requirements
for the original registration of the subject property. A party dealing with a
registered land need not go beyond the Certificate of Title to determine the
true owner thereof so as to guard or protect his or her interest. Hence, ALI was
not required to go beyond what appeared in the transfer certificate of title in the
name of its immediate transferor. It may rely solely, as it did, on the correctness of
the certificate of title issued for the subject property and the law will in no way
oblige it to go behind the certificate of title to determine the condition of the
property. This is the fundamental nature of the Torrens System of land registration,
to give the public the right to rely upon the face of a Torrens certificate of title and to
dispense with the need of inquiring further. It cannot be again said that the issuance
of OCT No. 242 was a result of the registration decree of the Court of First Instance
of Rizal, pursuant to land registration proceedings in Case No. 976. In the absence of
proof to the contrary, OCT No. 242 and its derivatives, including ALI’s TCT No. T-
41262, enjoy the presumption of regularity and ALI need not allege or prove that its
title was regularly issued. That is precisely the nature of such a presumption, it
dispenses with proof. Rule 131, Section 3 of the Rules of Court provides:

Section 3. Disputable presumptions. — The following presumptions are satisfactory


if uncontradicted, but may be contradicted and overcome by other evidence:
xxxx

(m) That official duty has been regularly performed;

(n) That a court, or judge acting as such, whether in the Philippines or elsewhere,
was acting in the lawful exercise of jurisdiction;

(o) That all the matters within an issue raised in a case were laid before the court
and passed upon by it; and in like manner that all matters within an issue raised in a
dispute submitted for arbitration were laid before the arbitrators and passed upon
by them; x x x.

To overturn this legal presumption carelessly — more than 90 years since the
termination of the case — will not only endanger judicial stability, but also violate
the underlying principle of the Torrens system. Indeed, to do so would reduce the
vaunted legal indefeasibility of Torrens titles to meaningless verbiage. The
presumption of regularity enjoyed by the registration decree issued in Case No. 976
and OCT No. 242 includes the presumption that all the requisites for the issuance of
a valid title had been complied with. ALI need not allege or prove that a duly
approved survey plan accompanied the issuance of OCT No. 242 in 1950 because
it is presumed. It is the party who seeks to overcome the presumption who would
have the burden to present adequate and convincing evidence to the contrary. This,
petitioners did not even attempt to do. We cannot accept petitioners’ proposition
that they did not have the burden of proof of showing the irregularity of ALI’s title
since the burden of proof purportedly did not shift to them since no full-blown trial
was conducted by the RTC.

This specious argument deserves scant credit. Rule 131, Section 1 of the Rules of
Court provides:

Section 1. Burden of proof. — Burden of proof is the duty of a party to present


evidence on the facts in issue necessary to establish his claim or defense by the
amount of evidence required by law.

With the filing of the complaint, petitioners should already have alleged all the bases
of their cause of action, particularly their allegation that ALI’s title is null and void
and that such title should be cancelled. However, a scrutiny of the complaint would
show that petitioners never alleged the purported lack of an approved survey plan
as a defect of ALI’s title.

Indubitably, in view of the CA’s Decision in CA-G.R. SP No. 44243, this controversy
has been reduced to the sole substantive issue of which between the two titles,
purporting to cover the same property, deserves priority. This is hardly a novel
issue. As petitioners themselves are aware, in Realty, it was held that:

In this jurisdiction, it is settled that "(t)he general rule is that in the case of two
certificates of title, purporting to include the same land, the earlier in date
prevails xxx. In successive registrations, where more than one certificate is issued
in respect of a particular estate or interest in land, the person claiming under the
prior certificate is entitled to the estate or interest; and that person is deemed to
hold under the prior certificate who is the holder of, or whose claim is derived
directly or indirectly from the person who was the holder of the earliest
certificate issued in respect thereof x x x."

In all, we find that the CA committed no reversible error when it applied the
principle "Primus Tempore, Portior Jure" (First in Time, Stronger in Right) in this
case and found that ALI’s title was the valid title having been derived from the
earlier OCT.Petitioners contend that it is error on the part of the CA to rule that their
cause of action has been barred by prescription and laches. According to them, since
the OCT from which ALI derived its title is void for want of a duly approved survey
plan, their cause of action did not prescribe. However, as discussed above, the
conclusion of the trial court that OCT No. 242 is void was not sufficiently borne out
by the evidence on record. Verily, the premise upon which petitioners build their
theory of imprescriptibility of their action did not exist. It appears that Lots 2 and 3
were adjudicated to Guico on the basis of Psu-80886 (Lot 3 is the subject matter of
the instant case), Lot 10 in favor of Baltazar on the basis of Psu 56007, under which
plaintiffs-appellees’ title was based, and the rest to the heirs of Narciso Mayuga.
While Baltazar claimed Lot 3 on the basis of his Psu-56007, his claim was rejected
and the Lot was adjudicated to Guico on the basis of his Psu-80886. It is clear,
therefore, that whatever claim plaintiffs-appellees have on the subject property on
the basis of Lot 3 Psu-56007, through their predecessor-in-interest, Florentino
Baltazar, the same had been clearly and finally denied by the Supreme Court in
Guico vs. San Pedro.

In conclusion, we find that the Court of Appeals committed no reversible error in


setting aside the patently erroneous Summary Judgment of the trial court.

DISPOSITIVE PORTION:
WHEREFORE, the petition is DENIED. The Court of Appeals’ Decision dated
December 22, 2003 and the Resolution dated December 16, 2004 are hereby
AFFIRMED.
THE DIRECTOR OF LANDS V. CA and TEODORO ABISTADO, G.R. No. 102858

Facts:

Teodoro Abistado, private respondent, Filed a petition for original registration of his
title over 648 square meters of land under P.D. No. 1529 or the Property
Registration Decree.

The application was docketed as Land Registration Case (LRC) No. 86 and assigned
to Branch 44 of the Regional Trial Court of Mamburao, Occidental Mindoro. During
the pendency of the case, Teodoro Abistado died and was substituted by his
children - Margarita, Marissa, Maribel, Arnold, and Mary Ann, all surnamed
Abistado, who were all represented by their aunt Josefa Abistado, ad litem ( act in
which a lawsuit has a representative in behalf of children not capable of
representation.) Land Registration Court dismissed the petition for want of
jurisdiction in compliance with the mandatory provision requiring publication of
initial public hearing in a newspaper of general circulation. Records show that
applicants failed to comply with P.D. No. 1529 Section 23 (1) requiring publication
of notice of initial hearing in a newspaper of general circulation. Initial public
hearing was only published in the Offiffifficial Gazette. The case was elevated to the
Court of Appeals which granted the application and ordered the registration of title
to Teodoro Abistado, since publication in a newspaper of general Circulation is
merely procedural, hence dispensable. The Director of Land, represented by the
Solicitor General, elevated this case to the Supreme Court.

Issue: Whether or Not the Director of Land is correct that the publication of Notice
of Initial hearing in a Land Registration Case is mandatory.

Held:

Yes. Section 23 of P.D. No. 1529 shall be followed requiring a publication once both
in the Offiffifficial Gazette and newspaper of general circulation. The Land
Registration Case is an in Rem proceeding, meaning the applicant must prove his
title over the land against all persons concerned, who might have interest to right in
the property and should effectively be invited in the court to prove why the title
should not be granted.

Such provision used the term "shall" which indicated that it is mandatory. When the
law speaks in clear and categorical language, there is no room for interpretation,
vacillation, or equivocation, there is room only for application.

Thus. Supreme Court affiffiffirmed the decision of the Lower Court dismissing the
petition for registration of Land Title to the respondents.
Republic vs. dela Paz GR No. 171631: November 15, 2010

Facts:

Respondents filed a case to register a parcel of land covering almost 25,800 square
kilometres situated in Taguig. They alleged that they came into the possesion of the
land thru their parents who have been continous, uninterrupted, open, public,
adverse possession of the same, in the concept of owner since 1987. The Republic
Petitioner opposed the application on the ground that they have not been in
continous, uninterrupted, open, public, adverse possesion of the same, in the
concept of owner but the RTC ruled in favor of the respondents. The Republic
interposed an appeal to the CA, but it was also denied, the courts reasoninh that
respondents have established their right to the parcel of land.

Issue:
1. whether or not the continuous, uninterrupted, open, public and adverse
possession are sufficiently established by evidence.

2. whether the land is part of the alienable part of public domain.

Ruling:

No, it has not been sufficiently established.

Civil Law Land Registration


1st issue: Respondents need to prove that (1) the land reforms part of the alienable
and disposable land of the public domain; and (2) they, by themselves or through
their predecessors-in-interest, have been in open, continous, exclusive, and
notorious possesion and occupation of the subject land under a bona fide claim of
ownership from June 12, 1945 or earlier. Respondents have not presented tangible
proof to establish this kind of possesion. At best, they have only given a tax
declaration on 1949, but this is merely indicia of ownership.

2nd issue: To overcome this presumption, incontrovertible evidence must be


established that the land subject for application (or claim) is alienable or
disposable.To support this, Respondents have merely relied on the survey plan of a
geodetic engineer. This is insufficient under the law. Respondents failed to submit a
certification from the proper government agency to establish that the subject land
arepart of the alienable and disposable portion of the public domain.
Republic vs. CA and Lapina

Facts:

On June 1978, respondent spouses bought from one Cristela Dazo Belen Lots
347 and 348, Cad. S38-D with a total land area of 91.77 sqm situated in San Pablo
City. At the time of the purchase, respondent spouses were then natural-born
Filipinos. On February 1987, the spouses filed an application for registration of title
of the 2 parcels of land before the RTC of San Pablo City. This time, however, they
were no longer Filipino citizens and have acquired Canadian citizenship through
naturalization.

Both the RTC and the CA confirmed the respondents’ title to the lots in
question. The petitioner submits that the respondents have not acquired
proprietary rights over the subject properties before they acquired Canadian
citizenship. It maintains that even privately owned, unregistered lands are
presumed to be public lands under the principle that land of whatsoever
classification belong to the state under the Regalian Doctrine. Thus, before the
certificate of title was issued, the occupant is not in the juridical sense the true
owner of the land since it still pertains to the State.

Issue:

Whether or not a foreign national (former Filipino) may validly apply for
registration of title over a parcel of land which he acquired when he was still a
Filipino citizen

Ruling:

Regardless of your citizenship upon registration, what is only necessary is


that you are a Filipino when you acquired the land as the owner. Under Sec 48 of CA
141, the law tacks possession over the property from predecessor-in- interest. It
does not matter whether the applicant has been in possession of the subject
property for only a day, so long as the period and or legal requirements for
confirmation of title has been complied with by his predecessors-in-interest. Since
the Lapina’s predecessors-in-interest have been shown to have been in open,
continuous, exclusive, and notorious possession and occupation over the land since
1937, the land, therefore, became private land and therefore registrable.

Registration is not a mode of acquiring ownership, but merely a formality


aimed to confirm a title which one already has. The Lapinas, therefore, had no legal
impediments to register the land which they validly and legally acquired while they
were Filipinos.
Republic vs. Espinosa

Facts:

On March 3, 1999, respondent Domingo Espinosa filed with the Municipal


Trial Court of Consolacion, Cebu an application for land registration covering a
parcel of land with an area of 5,525 square meters and situated
in Barangay Cabangahan, Consolacion, Cebu. In support of his application, Espinosa
alleged that: (a) the property, which is more particularly known as Lot No. 8499 of
Cad. 545-D (New), is alienable and disposable; (b) he purchased the property from
his mother, Isabel Espinosa (Isabel), on July 4, 1970 and the latter's other heirs had
waived their rights thereto; and (c) he and his predecessor-in-interest had been in
possession of the property in the concept of an owner for more than 30 years.

Espinosa submitted the blueprint of Advanced Survey Plan 07-000893 to


prove the identity of the land. As proof that the property is alienable and disposable,
he marked as evidence the annotation on the advance survey plan made by Cynthia
L. Ibañ ez, Chief of the Map Projection Section. He also presented 2 tax declarations
in Isabel's name to prove that she had been in possession of the property since
1965. To support his claim that he had been religiously paying the taxes due on the
property, Espinosa presented a Certification dated December 1, 1998 issued by the
Office of the Treasurer of Consolacion, Cebu and three (3) tax declarations for the
years 1978, 1980 and 1985.

Petitioner opposed Espinosa's application, claiming that: (a) Section 48(b) of


Commonwealth Act No. 141 otherwise known as the "Public Land Act" (PLA) had
not been complied with as Espinosa's predecessor-in-interest possessed the
property only after June 12, 1945; and (b) the tax declarations do not prove that his
possession and that of his predecessor-in interest are in the character and for the
length of time required by law.

MTC granted Espinosa’s petition for registration. The CA affirmed the MTC’s
judgement when the petitioner appealed for the decision.

Ruling:

The court disagrees with the MTC and the CA’s decision.

Espinosa failed to prove that: (a) Isabel's possession of the property dated
back to June 12, 1945 or earlier; and (b) the property is alienable and disposable. On
the other hand, applying Section 14(2) of P.O. No. 1529, Espinosa failed to prove
that the property is patrimonial.

The lower courts concluded that Espinosa complied with the requirements of
Section 48(b) of the PLA in relation to Section 14(1) of P.D. No. 1529 based on
supposed evidence that he and his predecessor-ininterest had been in possession of
the property for at least thirty (30) years prior to the time he filed his application.
However, there is nothing on record showing that as of January 25, 1977 or prior to
the effectivity of P.D. No. 1073, he or Isabel had already acquired title by means of
possession and occupation of the property for thirty (30) years. On the contrary, the
earliest tax declaration in Isabel's name was for the year 1965 indicating that as of
January 25, 1977, only twelve (12) years had lapsed from the time she first came
supposedly into possession.
Republic vs. Nillas

Facts:

In 1997, the respondent seeks for the revival of the 1941 decision of the CFI
of Negros Oriental. The CFI, acting as a cadastral court, adjudicated several lots,
together with the improvements thereon, in favor of named oppositors who had
established their title to their respective lots and their continuous possession
thereof since time immemorial and ordered the Chief of the General Land
Registration Office, upon the finality of the decision, to issue the corresponding
decree of registration. The petitioner contends that the petition of the respondent is
barred by prescription or laches due to the lapse of time from 1941 up to 1997.

Issue:

Whether or not prescription or laches may bar a petition to revive a


judgment in a land registration case.

Ruling:

Ruling of RTC

Trial on the merits ensued. The RTC heard the testimony of Nillas and
received her documentary evidence. No evidence was apparently presented by the
OSG. On 26 April 2000, the RTC rendered a Decision finding merit in the petition for
revival of judgment, and ordering the revival of the 1941 Decision, as well as
directing the Commissioner of
the Land Registration Authority (LRA) to issue the corresponding decree of
confirmation and registration based on the 1941 Decision.

OSG appealed to CA

The OSG appealed the RTC Decision to the Court of Appeals, arguing in main
that the right of action to revive judgment had already prescribed. The OSG further
argued that at the very least, Nillas should have established that a request for
issuance of a decree of
registration before the Administrator of the LRA had been duly made.

Ruling of CA

In its Decision, the Court of Appeals reiterated that the provisions of Section
6, Rule 39 of the Rules of Court, which impose a prescriptive period for enforcement
of judgments by motion, refer to ordinary civil actions and not to "special"
proceedings such as land registration cases. The Court of Appeals also noted that it
would have been especially onerous to require Nillas to first request the LRA to
comply with the 1941 decision considering that it had been established that the
original records in the 1941 case had already been destroyed and could no longer be
reconstructed.

OSG appealed to SC

The OSG strongly argues that the principles of prescription and laches do
apply to land registration cases. The OSG notes that Article 1144 of the Civil Code
establishes that an action upon judgment must be brought within ten years from the
time the right of action accrues. Further, Section 6 of Rule 39 of the 1997 Rules of
Civil Procedure establishes that a final and executory judgment or order may be
executed on motion within five (5) years from the date of its entry, after which time
it may be enforced by action before it is barred by statute of limitations.

SC Ruling

The court affirmed the ruling of CA. Sec 6, Rule 39 provides:


“After a decision in a land registration case has become
final, it may not be enforced after the lapse of a period of
10 years, except by another proceeding to enforce the
judgment or decision. Authority for this theory is the
provision in the Rules of Court to the effect that judgment
may be enforced within 5 years by motion, and after five
years but within 10 years, by an action (Sec. 6, Rule 39)”

Rule 39, as invoked by the Republic, applies only to ordinary civil actions, not to
other or extraordinary proceedings not expressly governed by the Rules of Civil
Procedure but by some other specific law or legal modality such as land registration
cases. Unlike in ordinary civil actions governed by the Rules of Civil Procedure, the
intent of land registration proceedings is to establish ownership by a person of a
parcel of land, consistent with the purpose of such extraordinary proceedings to
declare by judicial fiat a status, condition or fact. Hence, upon the finality of a
decision adjudicating such ownership, no further step is required to effectuate the
decision and a ministerial duty exists alike on the part of the land registration court
to order the issuance of, and the LRA to issue, the decree of registration.
Ting vs. Heirs of Lirio

Facts:

In a Decision of December 10, 1976 in Land Registration Case (LRC) No. N-


983, then Judge Alfredo Marigomen of the then Court of First Instance of Cebu,
Branch 7, granted the application filed by the Spouses Diego Lirio and Flora Atienza
for registration of title to Lot No. 18281of the Cebu Cadastral 12 Extension, Plan Rs-
07-000787. The decision in LRC No. N-983 became final and executory on January
29, 1977. Judge Marigomen thereafter issued an order of November 10, 1982
directing the Land Registration Commission to issue the corresponding decree of
registration and the certificate of title in favor of the spouses Lirio. On February 12,
1997, Rolando Ting (petitioner) filed with the Regional Trial Court (RTC) of Cebu an
application for registration of title to the same lot. The application was docketed as
LRC No. 1437-N.

The herein respondents, heirs of Diego Lirio, who were afforded the
opportunity to file an opposition to petitioner's application by Branch 21 of the
Cebu RTC, filed their Answer calling attention to the December 10, 1976 decision in
LRC No. N-983 which had become final and executory on January 29, 1977 and
which, they argued, barred the filing of petitioner's application on the ground of res
judicata. After hearing the respective sides of the parties, Branch 21 of the Cebu
RTC, on motion of respondents, dismissed petitioner's application on the ground of
res judicata.

Petitioner argues that although the decision in LRC No. N-983 had become
final and executory on January 29, 1977, no decree of registration has been issued
by the Land Registration Authority (LRA); it was only on July 26, 2003 that the
"extinct" decision belatedly surfaced as basis of respondents' motion to dismiss LRC
No. 1437-N; and as no action for revival of the said decision was filed by
respondents after the lapse of the ten-year prescriptive period, "the cause of action
in the dormant judgment passed into extinction." Petitioner thus concludes that an
"extinct" judgment cannot be the basis of res judicata.

Ruling:

The RTC dismissed Ting’s application on the ground of res judicata. Sec. 30 of
the Property Registration Decree provides: “x x x the judgment rendered in a land
registration proceeding becomes final upon the expiration of thirty days to be
counted from the date of receipt of notice of the judgment. An appeal may be taken
from the judgment of the court as in ordinary civil cases. x x x” The decision in a land
registration case, unless the adverse or losing party is in possession, becomes final
without any further action, upon the expiration of the period or perfecting an
appeal.
In a registration proceeding instituted for the registration of a private land,
with or without opposition, the judgment of the court confirming the title of the
applicant or oppositor, as the case may be, and ordering its registration in his name
constitutes, when final, res judicata against the whole world. It becomes final when
no appeal within the reglementary period is taken from a judgment of confirmation
and registration. Furthermore, there is no provision in the Land Registration Act
similar to Sec. 6, Rule 39, regarding the execution of a judgment in a civil action,
except the proceedings to place the winner in possession by virtue of a writ of
possession.

The decision in a land registration case, unless the adverse or losing party is
in possession, becomes final without any further action, upon the expiration of the
period. Section 30 of Presidential Decree No. 1529 or the Property Registration
Decree provides that after judgment has become final and executory, it shall devolve
upon the court to forthwith issue an order in accordance with Section 39 of this
Decree to the Commissioner for the issuance of the decree of registration and the
corresponding certificate of title in favor of the person adjudged entitled to
registration. The land registration proceedings being in rem, the land registration
court’s approval of spouses Diego Lirio and Flora Atienza’s application for
registration of the lot settled its ownership and is binding on the whole world
including petitioner.
Republic v. Claro Yap

Facts:
In 2010, respondent Claro Yap filed a petition for cancellation and re-
issuance of Decree No. 99500 which was issued in 1920 and which covers Lot No.
922 of the Carcar Cadastre, and for the issuance of the corresponding Original
Certificate of Title pursuant to the re-issued decree. The RTC, finding that Yap had
sufficiently established his claims and was able to prove his ownership and
possession over said lot, granted the petition and ordered the Register of Deeds of
the Province of Cebu to cancel Decree No. 99500, re-issue a new copy thereof, and
on the basis of such new copy, issue an OCT in the name of Andres Abellana, as
administrator of the Estate of Juan Rodriguez.

Since the order of the RTC was for the re-issuance of the decree under the
name of its original adjudicate, Yap filed a Partial Motion for Reconsideration stating
that the new decree and OCT should be issued under his name instead of Andres
Abellana. On the other hand, the Republic, through the Office of the Solicitor General,
filed its Comment arguing that Yap's petition and motion should be denied since the
Republic was not furnished with copies thereof. The RTC denied Yap's motion ruling
that the law provides that the decree, which would be the basis for the issuance of
the OCT, should be issued under the name of the original adjudicate. The RTC also
denied the OSG's motion finding that the records of the case show that it was
furnished with copies of the Petition as well as the Partial Motion for
Reconsideration.

The OSG then interposed an appeal before the CA arguing that Yap's petition
should have been denied due to insufficiency of evidence and failure to implead
indispensable parties such as the heirs of Juan Rodriguez and/or Andres Abellana.
The CA upheld the RTC's ruling finding that the pieces of evidence submitted by Yap
were sufficient to support the petition. As regards the OSG's argument on non-
joinder of indispensable parties, the CA highlighted that it is not a ground for
dismissal of an action. Nevertheless, it ruled that the heirs of either Andres Abellana
or Juan Rodriguez were not deprived of the opportunity to be heard as the
proceeding before the RTC was an in rem proceeding. Thus, when the petition was
published, all persons including the said heirs were deemed notified.

Issue:

Whether the RTC correctly ordered the cancellation of Decree No. 99500, the
re-issuance thereof, and the issuance of the corresponding Original Certificate of
Title covering Lot No. 922.

Ruling:

YES. Settled is the rule that prescription cannot be raised for the first time on
appeal; the general rule being that the appellate court is not authorized to consider
and resolve any question not properly raised in the courts below. In any event,
prescription does not lie in the instant case.

Records show that Yap sufficiently established that Decree No. 99500 was
issued in 1920 in the name of Andres Abellana, as Administrator of the Estate of
Juan Rodriguez. Further, it was also proven during the proceedings before the court
that no OCT was ever issued covering the said lot. In this regard, Section 39 of
Presidential Decree No. 1529 or the "Property Registration Decree" provides that
the original certificate of title shall be a true copy of the decree of registration. There
is, therefore, a need to cancel the old decree and a new one issued in order for the
decree and the OCT to be exact replicas of each other.

Based from the foregoing, the RTC correctly ordered the cancellation of
Decree No. 99500, the reissuance thereof, and the issuance of the corresponding
OCT in the name of its original adjudicate, Andres Abellana, as Administrator of the
Estate of Juan Rodriguez.
Manotok Realty vs. CLT Realty

Facts:

Both petitioner and respondent claim ownership over a parcel of land. Claim
of ownership sprung from OCT No. 994. The confusion arises because of the fact that
the petitioner’s OCT No. 994 was registered on May 3, 1917 while respondent’s OCT
994 was registered on April 19, 1917. Record shows that OCT 994 was received by
the Register of Deeds for Transcription of May 3, 1917.

Issue:

Which of the contending parties are able to trace back their claims of title to
OCT No. 994 dated 3 May 1917?

Ruling:

There is only one OCT 994, the registration date of which had already been
decisively settled as 3 May 1917 and not 19 April 1917. OCT 994 which reflects the
date of 19 April 1917 as its registration date is null and void. The certificates of title
of the deceased Jose Dimson and his successor-in-interest, CLT, having been traced
back to OCT 994 dated 19 April 1917, are NULL and VOID and thus vest no legal
right or claim in favor of CLT Realty.
The certificates of title covering the land being claimed by Araneta/Manotok
were derived from OCT NO. 994 registered on 3 May 1917 thereby ultimately
showing a direct link of TCT Nos. 7784 and 13574 to said mother title.

The land becomes a registered land only upon the transcription of the decree
in the original registration book by the Register of Deeds, and not on the date of
issuance of the decree. Otherwise stated, as soon as the decree has been registered
in the office of the Register of Deeds, the property described therein shall become
registered land, and the certificate shall take effect upon transcription of the decree.
Angeles v Sec. of Justice, 614 SCRA 478, March 9,2010

The property involved in this case is covered by Original Certificate of Title


No. 994, (1,342) hectares

On May 3, 1965, petitioner, together with other individuals, all of them


claiming to be the heirs of a certain Maria de la Concepcion Vidal, and
alleging that they are entitled to inherit her proportional share in the
parcels of land located in Quezon City and in the municipalities of Caloocan
and Malabon, Province of Rizal, commenced a special civil action for partition
and accounting special civil action for partition and accounting of the property
otherwise known as Maysilo Estate covered by OCT No. 994, allegedly
registered on April 19, 1917 with the Registry of Deeds of Caloocan City.

RTC granted the partition prayed for by the plaintiffs; directed the respective
Registers of Deeds of Caloocan City and Quezon City to issue transfer
certificate of title in the names of all the co-owners

However, registers of Deeds of Caloocan City and Quezon City refused to


comply with the RTC Order because they were still awaiting word from the
LRA Administrator before proceeding.

LRA said that, the mess caused by the former Register of Deeds and Deputy
Register of Deeds in making it appear that OCT No. 994 was issued in 19 April
1917, thus giving the wrong impression that there were two (2) OCT No. 994,
resulted in the double, if not multiple, issuance of transfer certificates of
title covering the subdivided portions of the Maysilo Estate, including the
parcels of land mentioned in the subject Order dated 8 January 1998. Our
Authority, as the protector of the integrity of the Torrens title is mandated to
prevent anomalous titling of real properties and put a stop to further erode
the confidence of the public in the Torrens system of land registration.

Also, the request cannot be granted in view of the directive of the Department
of Justice in its 1st Indorsement dated September 1997, as a result of the
inquiry conducted by the Composite FactFinding Committee, finding that there
is only one OCT No. 994 which was issued by the Rizal Register of Deeds
on 3 May 1917 (and not on 19 April 1919)

Petitioner then argued that they are entitled to mandamus as they have suffered
from damages and losses because of the wrongful act of the respondent.

However, the SC did not agree with the petitioner

The issuance by the LRA officials of a decree of registration is not a purely


ministerial duty in cases where they find that such would result to the double
titling of the same parcel of land.
From the findings, we can notice that the TCT dated April 19 1919 was in fact void
because of a fabrication perpetrated by the former Deputy Registrar of Deeds of
Caloocan City.

Specifically, petitioner cannot anymore insist that OCT No. 994 allegedly issued
on April 19, 1917 validly and actually exists, given the following conclusions
made by this SC in the 2007 Manotok case (3 years before this decision) ,
declaring the said TCT void.
Rep. Of the Phil v Sps. Go, GR No. 197297, Aug 2, 2017

Respondents Spouses applied for the registration and confirmation of title over
Cadastral Lot No. 4699-B , a parcel of land in Barangay Balagtas, Batangas City
covering an area of 1,000 square meters.

The Spouses Go registered Lot No. 4699-B in their names for taxation
purposes. They had paid the real property taxes, including the arrears, from
1997 to 2006, as shown in Tax Declaration No. 026-04167.

They had also established a funeral parlor, San Sebastian Funeral Homes, on
the lot. According to them, there were no other claimants over the property.
Spouses Go claimed to be in an open, continuous, exclusive, notorious, and
actual possession of the property for seven (7) years since they bought it.

They also tacked their possession through that of their predecessors-in-interest,


as follows: Sometime in 1945, Anselmo de Torres (Anselmo) came to know
that his parents, Sergia Almero and Andres de Torres (the Spouses de Torres),
owned Lot No. 4699, a bigger property where Lot No. 4699-B came from.
According to Anselmo, the Spouses de Torres paid the real property taxes
during their lifetime and planted bananas, mangoes, calamansi, and rice on this
lot. His mother, Sergia Almero (Sergia), allegedly inherited Lot No. 4699 from
her parents, Celodonio and Eufemia Almero (the Spouses Almero). In the
1960s, Anselmo and his siblings inherited Lot No. 4699 from their parents
upon their deaths.

On Jan 26, 2000, spouses Go bought Lot No. 4699-B from the previous owners
as evidenced by a Deed of Absolute Sale. During the application for the
registration, they attached the Report of Special Land Investigator(CENRO),
stating that the property was located in an alienable and disposable zone.

Republic of the Philippines opposed respondents' application for registration for


the following reasons: 1) Lot No. 4699-B was part of the public domain; 2)
neither the Spouses Go nor their predecessors-in interest had been in open,
continuous, exclusive, and notorious possession and occupation of the property
since June 12, 1945 or even before then; 3) the tax declaration and payment
were not competent or sucient proof of ownership, especially considering that
these were relatively recent.

MTC rendered a decision in favor of the Spouses

CA denied the appeal of the Rep

SC grant such petition by the Rep


In an application for judicial confirmation of title, an applicant already holds
an imperfect title to an agricultural land of the public domain after having
occupied it from June 12, 1945 or earlier. Thus, for purposes of obtaining an
imperfect title, the date it was classified is immaterial.

The land may be declared alienable and disposable at any time, not necessarily
before June 12, 1945. The moment that the land is declared alienable and
disposable, an applicant may then initiate the proceedings for the judicial
confirmation of title.

On the other hand, for the requisite duration of possession, an applicant must
have had possession of the property under a bona de claim of ownership or
acquisition, from June 12, 1945 or earlier. Such possession must have also
been open, continuous, exclusive, and notorious.

The Spouses Go's possession, by themselves or through their predecessors in


interest, does not meet the statutory requirements. The evidence the Spouses
Go submitted to prove their required length of possession consist of Anselmo's
testimony, Cristina's sole Tax Declaration, and the Spouses Go's sole Tax
Declaration. Other than these pieces of evidence, the Spouses Go could not
support their claim of possession in the concept of an owner, by themselves or
through their predecessors-in-interest, from June 12, 1945 or earlier.

Even assuming that there is sufficient evidence to establish their claim of


possession in the concept of an owner since June 12, 1945, the Spouses Go
nevertheless failed to prove the alienable and disposable character of the land.

T]he exclusive prerogative to classify public lands under existing laws is vested
in the Executive Department.

The CENRO Certification, which they attached during application, should have
been accompanied by an official publication of the DENR Secretary's
issuance declaring the land alienable and disposable. A CENRO certification,
by itself, is insufficient to prove the alienability and disposability of land
sought to be registered.

The pieces of evidence the Spouses Go adduced fall short of the requirements
of the law.
Rep v Sps. Noval, GR No. 170316, Sept. 18, 2017

Applicants sought the registration of their titles over the subdivided portions
of a land in Barangay Casili, Consolacion, Cebu, . They alleged to have acquired
their respective portions of this land by "purchase, coupled with continuous,
public, notorious, exclusive and peaceful possession in the concept of an owner
for more than 30 years including [the possession] of their predecessors-in-
interest." They also alleged that they were in actual possession of their
respective portions of the property.

The Republic filed its Opposition on the ground that the applicants failed to
prove open, continuous, exclusive, and notorious possession of the property
since June 12, 1945. It also argued that the property sought to be registered
was part of the public domain. It alleged that the tax declarations and tax
payment receipts attached to the application were not competent to show bona
fide acquisition or open and continuous possession of the land.

The Municipal Trial Court granted their application for registration of title. It
declared the applicants to be the absolute owners and possessors of their
respective lots, having established conclusively that they are the exclusive
owners and peaceful possessors of the properties. The trial court ordered the
issuance of decrees of registration upon finality of its judgment.

CA affirmed

The Court of Appeals also found that while the applicants did not submit a
Department of Environment and Natural Resources report showing that the
property had been declared alienable and disposable, the Republic was not
relieved of the duty to present evidence that the land belongs to the
public domain. It ruled that the burden is upon the State to prove that land
is public domain when it has been possessed and cultivated by an applicant
and his or her predecessors-in-interest for a considerable number of years
without action from the State.

SC denied the petition of the Rep

Applicants for judicial confirmation of title must still comply with the
requisites stated in Section 48 (b) of the Public Land Act and Section 14 (1)
of the Property Registration Decree:
1. The applicant, by himself or through his predecessor-in-interest, has
been in possession and occupation of the property subject of the
application;
2. The possession and occupation must be open, continuous, exclusive, and
notorious;
3. The possession and occupation must be under a bona de claim of
acquisition of ownership;
4. The possession and occupation must have taken place since June 12,
1945, or earlier; and
5. The property subject of the application must be an agricultural land of
the public domain.
In order to establish that an agricultural land of the public domain has
become alienable and disposable, "an applicant must establish the existence of
a positive act of the government such as a presidential proclamation or an
executive order; an administrative action; investigation reports of Bureau of
Lands investigators; and a legislative act or a statute." However, When the
State has no effective opposition, except for a pro forma opposition, to
controvert an applicant's convincing evidence of possession and occupation,
presumptions are tilted to this applicant's favor.

The burden of evidence lies on the party who asserts an affirmative allegation.
Therefore, if the State alleges that lands belong to it, it is not excused from
providing evidence to support this allegation. This specially applies when the
land in question has no indication of being incapable of registration and has
been exclusively occupied by an applicant or his or her predecessor-in-interest
without opposition — not even from the State.

When an applicant in the registration of property proves his or her open,


continuous, exclusive, and notorious possession of a land for the period
required by law, he or she has acquired an imperfect title that may be
confirmed by the State. The State may not, in the absence of controverting
evidence and in a pro forma opposition, indiscriminately take property without
violating due process.
Kawayan Hills Corp v CA, GR No. 203090, Sept. 5, 2018

Kawayan Hills is a domestic corporation dealing with real estate. KHC filed an
application for confirmation and registration of Lot No. 2512's title in its
name before the Municipal Circuit Trial Court of Paoay-Currimao. They claimed
to have acquired Lot. No. 2512 on Dec. 27, 1995 through a Deed of Adjudication
with Sale executed by Servando Teolo and Maria Dafun, the successors-in-
interest of Andres Dafun (Andres). Andres had been Lot No. 2512's real
property tax declarant since 1931. Andres, with his eight (8) children, had also
allegedly possessed, cultivated, and harvested Lot No. 2512's fruits.

Kawayan Hills submitted the following documents in support of its application:


1. Certificate of Incorporation of Kawayan Hills Corporation
2. Secretary's Certificate
3. Tax Declaration No. ARP No. 96-025-02624
4. Deed of Adjudication with Sale dated 27 December 1995
5. Municipal Treasurer Certificate of Non-Tax Delinquency
6. BIR Certificate Authorizing Registration of Documents
7. Municipal Treasurer Certificate that applicant was a real property taxpayer
8. DENR Certificate re: within disposable and alienable lands
9. DENR Certificate re: not identical to previously approved isolated survey
10. DAR Order of Exemption dated 28 March 2001
11. Technical Description
12. Survey/Issuance Plan of Lot 2512 (Ap-01-004666)

The Republic of the Philippines, filed its Opposition to the application. It


asserted that Kawayan Hills failed to comply with the requirements of Section
14 (1) of Presidential Decree No. 1529, otherwise known as the Property
Registration Decree, for judicial confirmation of imperfect title.

MTC ruled in favor of KHC

CA reversed, its argues is that the real property tax declarations are not
conclusive evidence of ownership demonstrates its failure to go about its duty
of resolving the case with care and precision. It indicates grave abuse of
discretion.

SC ruled in favor of KHC

[T]he applicant must satisfy the following requirements in order for his
application to come under Section 14 (1) of the Property Registration Decree,
to wit:
1. The applicant, by himself or through his predecessor-in-interest, has
been in possession and occupation of the property subject of the
application;
2. The possession and occupation must be open, continuous, exclusive, and
notorious;
3. The possession and occupation must be under a bona de claim of
acquisition of ownership;
4. The possession and occupation must have taken place since June 12,
1945, or earlier; and
5. The property subject of the application must be an agricultural land of
the public domain.

The CA erred in reversing the decision, they even conceded that the first of its
identified requisites is availing here. For the second requisite, they also conceded
by saying “Kawayan Hills had likewise met the second requirement as to
ownership and possession. The [Municipal Circuit Trial Court] found that it
had presented sufficient testimonial and documentary evidence to show that from
its first known predecessor-in-interest, Andres Dafun, up to [itself], they were in
open, continuous, exclusive and notorious possession and occupation of the land
in question.

As to the third requisite, CA decried petitioner's reliance on tax declarations, even


if they dated to as far back as 1931, as these supposedly did not prove
ownership.

The SC said that tax declarations do not absolutely attest to ownership, this
Court has also recognized that "[t]he voluntary declaration of a piece of
property for taxation purposes . . . strengthens one's bona de claim of
acquisition of ownership." It has stated that payment of real property taxes
"is good indicia of possession in the concept of an owner, and when coupled
with continuous possession, it constitutes strong evidence of title."

From the documented possession in 1931, no one has come forward to contest
his and his successors-in-interest's possession as owners. It was only on
September 4, 2001, about a month after petitioner's ling of its application, that
the Republic came forward to contest the confirmation and registration of title
in his name.

A court confronted with an application for judicial confirmation of imperfect


title cannot casually rely on the expedient formula that real property tax
declarations are not conclusive evidence of ownership as a catch-all key to
resolving the application. Instead, it must carefully weigh competing claims and
consider the totality of evidence, bearing in mind the recognition in
jurisprudence that payment of real property taxes is, nevertheless, "good
indicia of possession in the concept of an owner, and when coupled with
continuous possession, it constitutes strong evidence of title.
Rep v Javier, GR No. 214367, April 4, 2018

This case is about the respondents’ application for registration of land title over a
parcel situated in Barangay Tranca, Talisay, Batangas led in June 2009 before
the Municipal Circuit

Rep, likewise, filed an opposition that the respondent did not met the requirements
as specified by law especially the certification from the DENR which should classify
the land as alienable and disposable.

The respondent only filed, in their application, the document which was certified by
CENRO w/o any certification from the DENR.

However, the lower courts still rendered a decision in favor of the respondent

The SC did not agree with the lower court

To establish that the land sought to be registered is alienable and disposable,


applicants must "present a copy of the original classification approved by the
[Department of Environment and Natural Resources] Secretary and certified as
a true copy by the legal custodian of the official records."

The certification issued by the DENR Secretary is necessary since he or she is


the official authorized to approve land classification, including the release of
land from public domain.

In this case, although respondents were able to present a CENRO certification,


and the survey plan showing that the property is already considered alienable
and disposable, these pieces of evidence are still not sufficient to prove that
the land sought to be registered is alienable and disposable. Absent the DENR
Secretary's issuance declaring the land alienable and disposable, the land
remains part of the public domain.

Thus, even if respondents have shown, through their testimonial evidence, that
they and their predecessors-in-interest have been in open, continuous,
exclusive, and notorious possession and occupation of the property since June
12, 1945, they still cannot register the land for failing to establish that the
land is alienable and disposable
Heirs of Delfin v NHA, GR No. 193618, Nov 28, 2016

Delfin Spouses claimed that they were the owners of a 28,800 square meter
parcel of land in Townsite, Suarez, Iligan City. As their evidence, the furnished
certifications and letters from government agencies, which support their claims,
particularly, their and their predecessors-in-interest's possession since June 12,
1945

They further alleged that, sometime in 1982, respondent National Housing


Authority forcibly took possession of a portion of the property. Despite their
repeated demands for compensation, the National Housing Authority failed to
pay the value of the property. The Delfin Spouses thus, filed their Complaint

NHA alleged that the Spouses' property was part of a military reservation area.
It cited Proclamation No. 2151 as having supposedly reserved the area in
which property is situated for Iligan City's slum improvement and resettlement
program, and the relocation of families who were dislocated by the National
Steel Corporation's five-year expansion program.

NHA failed to appear during the pre-trial conference. Upon the Spouses'
motion, the Regional Trial Court declared the National Housing Authority in
default.
Thus rendering a decision in favor of the Spouses

CA reversed

SC granted the petition of the spouses

Whatever rights petitioners may have had over the Iligan property was, thus,
not obliterated by Proclamation No. 2151. On the contrary, the Proclamation
itself facilitated compensation.

Section 48 (b) of the Public Land Act therefore requires that two (2)
requisites be satisfied before claims of title to public domain lands may be
confirmed: first, that the land subject of the claim is agricultural land; and
second, open, continuous, notorious, and exclusive possession of the land since
June 12, 1945.

First, there is no issue that the Iligan Property had already been declared to
be alienable and disposable land. Respondent has admitted this and Deputy
Public Land Inspector Pio Lucero, Jr.'s letters to the Director of Land attest to
this.

Second, although the Spouses' testimonial evidence and tax declarations


showed that their possession went only as far back as 1952, Deputy Public
Land Inspector Pio Lucero, Jr.'s letters to the Director of Land nevertheless
attest to a previous finding that the property had already been occupied as
early as June 1945.

Having shown that the requisites of Section 48 (b) of the Public Land Act
have been satisfied and having established their rights to the Iligan Property, it
follows that petitioners must be compensated for its taking because, there is a
right to be justly compensated whenever private property is taken for public
use as per Article III, Section 9 of the 1987 Constitution

You might also like